De Virgilio

Lakukan tugas rumah & ujian kamu dengan baik sekarang menggunakan Quizwiz!

16. Which of the following ventilator settings or respiratory measurements would be LEAST acceptable for a newborn with congenital diaphragmatic hernia (CDH)? (A) 100% FiO2 (B) PEEP of 20 cm H2O (C) Inhaled nitric oxide (NO) (D) PaCO2 of 55 mmHg (E) PaO2 of 65 mmHg

(A) 100% FiO2 *(B) PEEP of 20 cm H2O (C) Inhaled nitric oxide (NO) (D) PaCO2 of 55 mmHg (E) PaO2 of 65 mmHg

16. In a patient with claudication, the 5-year risk of limb loss is about: (A) 2-5 % (B) 8-10 % (C) 20-25 % (D) 40-50 % (E) 70-80 %

(A) 2-5% note: five year mortality in claudicants, however, is significantly increased as compared to non-claudicants, predominantly due to increased risk of MI (and to lesser degree, stroke)

10. A 29-year-old woman arrives to the ED following a jet ski accident. She has multiple bruises on her head, torso, and legs. She did not lose consciousness. Her temperature is 99.8°F, blood pressure is 108/78 mmHg, and pulse is 102/min. She only opens her eyes when you speak loudly to her. She is confused and screams in pain and withdraws when you palpate her left lower leg. Her imaging is pending. What is her calculated Glasgow Coma Scale? (A) 6 (B) 8 (C) 10 (D) 12 (E) 14

(A) 6 (B) 8 (C) 10 *(D) 12 (E) 14

9. A 38-year-old presents with symptoms and signs of acute appendicitis and undergoes laparoscopic appendectomy. At surgery the terminal ileum and cecum appear to be red and inflamed. The appendix is removed uneventfully. Final pathology of the appendix demonstrates no evidence of acute appendicitis. Two weeks later, he presents back to the ED with feces draining from his right lower quadrant wound. Which of the following is the most likely explanation for why the drainage may not spontaneously stop?

(A) A distal colonic obstruction *(B) Chronic inflammation (Crohn's or 'regional enteritis' can often be confused as appendicitis. HIS FRIEND acronym for things that prevent fistulas from closing) (C) An occult intra-abdominal abscess (D) A retained sponge in the abdomen (E) A missed malignancy

10. Which of the following is the next best step in the management of a 1-week-old infant born at full term with bilious emesis? (A) Abdominal ultrasound (B) Broad-spectrum antibiotics and blood cultures (C) Immediate operative repair (D) Contrast enema (E) IV fluids and nasogastric tube placement

(A) Abdominal ultrasound (B) Broad-spectrum antibiotics and blood cultures (C) Immediate operative repair (D) Contrast enema *(E) IV fluids and nasogastric tube placement

1. A 55-year-old male arrives to the ED with 40 % total body surface area second and third degree burns over his arms and legs after hot tar spilled on him at a jobsite where he was working as a roofer. He is in critical condi- tion and intubated in the ICU. On the fifth hospital day, he spikes a temperature of 102 °F, blood pressure is 110/80 mmHg, and pulse is 92/min. On physical exam he has tenderness on palpation of the RUQ and epigastrium, absent bowel sounds, and multiple healing burn wounds that appear to be clean. Laboratory exam demonstrates a WBC of 16×103/μL (normal 4.1-10.9×103/μL) with 12 % bands, amylase of 180 μ/L (normal 30-110 μ/L), lipase of 55 μ/L (7-60 μ/L), alkaline phosphatase of 70 μ/L (33-131 μ/L), and total bilirubin of 1.2 (0.1- 1.2 mg/dl). An abdominal X-ray series reveals distended loops of small bowel and large bowel without air fluid levels and no free air under the diaphragm. An abdominal ultrasound demonstrates a distended gallbladder with pericholecystic fluid and no stones. What is the most likely underlying etiology?

(A) Acute pancreatitis *(B) Cholecystitis (acalculous) (C) Cholangiohepatitis (D) Perforated duodenal ulcer (E) Acute cholangitis

2. A 55-year-old woman is evaluated in the emergency department for a 2-day history of severe epigastric abdominal pain, nausea, and vomiting. In reviewing her past medical history, she states she was evaluated 6 months ago for mild, but similar intermittent abdominal pain and was lost to follow-up. She does not take any medications. She has 1-2 drinks of alcohol on social occa- sions. On physical examination, temperature is 99.2 °F, blood pressure is 132/82 mmHg, pulse is 101/min, and respirations are 20/min. There is epigastric tenderness and RUQ tenderness. Labs are drawn and shown below. What is the most likely diagnosis? AST: 523 μ/L (normal 5-35 μ/L) ALT: 622 μ/L (7-56 μ/L) TBilli: 2.0 mg/dL (0.1-1.2 mg/dL) Alkaline phosphatase: 450 μ/L (33-131 μ/L) Amylase: 1300 μ/L (30-110 μ/L) Lipase: 1000 μ/L (7-60 μ/L)

(A) Acute pancreatitis secondary to alcohol *(B) Acute pancreatitis secondary to gallstones (C) Acute pancreatitis secondary to hypertriglyceridemia (D) Acute pancreatitis secondary to hypercalcemia (E) Chronic pancreatitis

12. A 63-year-old woman with diabetes is recovering in the ICU after receiving a CABG for coronary artery disease. On the sixth postoperative day, she starts complaining of chest pain. Her temperature is 101.4 °F, blood pressure is 108/72 mmHg, and pulse is 125/min. On physical exam, there is drainage from her sternal wound, and there is a crunching sound heard with a stethoscope over the precordium during systole. The sternum feels somewhat unstable to palpation. Her laboratory examination is significant for an elevated white blood count (16.7 × 103/μL). What is the most likely diagnosis?

(A) Acute pericarditis (B) Postoperative MI (C) Empyema *(D) Acute mediastinitis (E) Pneumonia

3. A 66-year-old male presents with a large volume of maroon-colored stools combined with red blood. In the ED his blood pressure is 100/60 mmHg, and heart rate is 120/min. Physical examination is unremarkable. Two large bore IVs are inserted, and 2 liters of normal saline are given, after which the patient's vital signs normalize. Laboratory tests are sent, including a type and cross. What is the next step in the management?

(A) Administer two units of O negative blood *(B) Place NG tube for aspiration (C) Colonoscopy (D) Exploratory laparotomy (E) Place central line

8. A 28-year-old woman presents to the ED because of abdominal pain that started 2 h ago. She has nausea and vomited twice in the last hour. She reports that the pain is predominantly in the right lower quadrant. On physical exam, her blood pressure is 120/60 mmHg, heart rate is 100/min, and she is afebrile. She is very tender in the RLQ on palpation at McBurney's point and has pain on passive extension of the right hip. Her WBC is 14 × 10 3 /μL (normal 4.1-10.9 × 10 3 /μL), hemoglobin is 12 g/dL (12-15.2 g/dL), hematocrit is 36 % (37-46%), and platelet count is 250,000 (140-450,000). Her urinalysis shows 1+ white blood cells. Which of the following is the best next step in management?

(A) Admit for observation (B) CT scan of the abdomen (C) Laparoscopic appendectomy *(D) B-hCG (E) Broad-spectrum antibiotics

20. A 1-week-old male presents with intolerance of breast- feedings as well as several episodes of bilious vomiting. On physical exam, the patient's blood pressure and temperature are normal. The infant appears to be in pain. However, the abdomen does not appear to be distended, and there is no tenderness to palpation. Plain abdominal x-ray shows an absence of gas within the bowel, but is otherwise unremarkable. A complete blood count and electrolytes are normal. What is the next step in the management?

(A) Admit for observation (B) Discharge patient and offer parents reassurance (C) Exploratory laparotomy *(D) Upper GI study with oral contrast (E) Abdominal ultrasound

8. A 20-year-old female arrives to the ED after slipping and hitting her head on ice and briefly losing consciousness. In the ED, she vomits twice. She denies amnesia. Her temperature is 98.9°F, blood pressure is 110/80 mmHg, and pulse is 80/min. Her GCS is 15. Her physical exam is normal with no papilledema. What is the most appropri- ate next step in management?

(A) Admit for observation and order CT scan of the head only if she develops a neurologic deficit (B) Admit for observation and start corticosteroids *(C) Order CT scan of the head without contrast now (D) Discharge home now (E) Discharge home with a tapered dose of corticosteroids

12. Which of the following is a risk factor for pancreatic cancer? (A) Alcohol (B) Smoking (C) Prostate cancer in the family (D) Malabsorption (E) Pancreatic enzyme supplementation

(A) Alcohol *(B) Smoking (C) Prostate cancer in the family (D) Malabsorption (E) Pancreatic enzyme supplementation

13. Where are melanomas in patients with dark skin most likely to occur? (A) Back (B) Arms (C) Legs (D) Palms, soles, and mucous membrane (E) Face

(A) Back (B) Arms (C) Legs *(D) Palms, soles, and mucous membrane (E) Face

7. A 50-year-old field worker arrives to a free clinic to discuss a "sore" on his lower lip. He has had no trauma to the face. He reports that he first noticed the "sore" 6 months ago, and it has slowly gotten bigger. On physical exam, he has an ulcerated 1 cm nodule on his lower lip. There are no telangiectasias present. What is the most likely diagnosis? (A) Basal cell carcinoma (B) Squamous cell carcinoma (C) Lichen planus (D) Dermatitis herpetiformis (E) Melanoma

(A) Basal cell carcinoma *(B) Squamous cell carcinoma (C) Lichen planus (D) Dermatitis herpetiformis (E) Melanoma

4. A 31-year-old breastfeeding female comes to the doctor for localized swelling, redness, and pain of the left breast. She also reports muscle aches and fatigue. On physical exam her temperature is 101.1 °F, pulse is 82/ min, blood pressure is 126/68 mmHg, and respirations are 16/min. Physical exam reveals a localized area of erythema and warmth in the left breast with no palpable masses. There is no axillary lymphadenopathy. What is the most likely next course of action?

(A) Biopsy *(B) Antibiotic treatment and continue breast feeding (tx for lactation mastitis) (C) Antibiotic treatment and encourage bottle-feeding only (D) Diagnostic mammography (E) Incision and drainage

5. A 50-year-old man comes to the doctor complaining of painful defecation. On occasion, blood is found on the toilet paper after wiping. He has been experiencing this problem for months but felt too embarrassed to seek medical attention. He has a history of constipation and has tried multiple stool softeners but to no avail. Rectal examination shows enlarged anal papillae with an edematous and hypertrophic skin tag in the posterior anal midline, most suggestive of a chronic fi ssure. He is referred to a surgeon and scheduled to undergo a lateral internal sphincterotomy. What is the most common complication of this procedure?

(A) Bleeding (B) Anorectal sepsis *(C) Fecal incontinence (D) Fistula formation (E) Nonhealing of fissure

16. A 67-year-old male is diagnosed with a type B aortic dissection. At the time of initial presentation on the previous day, his blood pressure was 178/110 mmHg. He was treated with intravenous beta-blocker, and his blood pressure was reduced to 112/60 mmHg and has remained in that range. However, one day later, he suddenly develops severe abdominal pain. His blood pressure is measured to be 110/56mmHg. Which of the following is the most likely explanation?

(A) C. difficile infection *(B) Occlusion of the superior mesenteric artery (C) Pancreatitis (D) Aortoenteric fistula (E) Diverticulitis

1. A 27-year-old woman has 3 months of intermittent spells of severe headache, heart palpitations, and sweating. A pregnancy test at her primary care doctor's office is positive. Further workup reveals that her plasma metanephrine level is 220 pg/ml (normal 12-60 pg/ml). What is the next step in establishing the diagnosis? (A) CT abdomen (B) Repeat plasma metanephrine level after the patient has delivered (C) MRI abdomen (D) I131-MIBG scan (E) Reassure patient that symptoms are related to pregnancy

(A) CT abdomen (B) Repeat plasma metanephrine level after the patient has delivered *(C) MRI abdomen (D) I131-MIBG scan (E) Reassure patient that symptoms are related to pregnancy

1. A 55-year-old male has been receiving serial ultrasound examinations to follow his abdominal aortic aneurysm (AAA). Over the past year, the aneurysm has rapidly enlarged to 5.8 cm, and he undergoes endovascular abdominal aortic aneurysm repair (EVAR). The operation itself is uneventful. However, on postoperative day 1, the patient develops a low-grade fever, left lower quadrant pain, and diarrhea that appears to be blood tinged. On physical examination, he has mild to moderate tenderness in the left lower quadrant without rebound or guarding. What is the next step in the workup?

(A) CT scan of the abdomen and pelvis with oral and IV contrast (B) Exploratory laparotomy (C) Formal mesenteric arteriography *(D) Flexible sigmoidoscopy- EVAR involves ligation of IMA (E) Abdominal ultrasound

19. A 45-year-old female presents with a 2 cm painless mass in her right anterior neck that has been present for 3 months and slowly enlarging. On physical exam, the mass feels firm and moves up and down with swallow- ing. She denies weight loss, weight gain, heat intolerance, or anxiety. A serum TSH level is normal. The most important step in the workup is: (A) CT scan of the neck (B) MRI of the neck (C) Fine-needle aspiration (FNA) (D) Open biopsy (E) Nuclear scan

(A) CT scan of the neck (B) MRI of the neck *(C) Fine-needle aspiration (FNA) (D) Open biopsy (E) Nuclear scan

11. A 22-year-old patient arrives to the ED after falling off his bicycle and hitting his head. In the ED, he is clearly intoxicated and his speech is slurred. He denies any neck pain but is not cooperative during the exam. He is placed in a cervical spine collar. What is the optimal method for clearance of the cervical spine in this patient?

(A) CT scan of the neck only (B) MRI of the neck only (C) MRI of the neck followed by a CT only if the MRI is negative *(D) CT scan of the neck followed by MRI only if the CT is negative (E) The C-spine can be cleared using physical examination

15. A 56-year-old woman is recovering after undergoing total thyroidectomy for papillary carcinoma. Her temperature is 99.8 °F, blood pressure is 120/80 mmHg, and pulse is 90/min. During her postoperative examination by the intern, the patient complains of numbness and tingling around her mouth and in her hands and feet. What could have been done postoperatively to anticipate and potentially remedy these symptoms?

(A) Check magnesium *(B) Check parathyroid hormone (C) Check potassium (D) Check TSH and free T4 (E) Carotid ultrasound

14. A 64-year-old male returns for follow-up six months after successful sigmoid colectomy for colon cancer. Which of the following can lead to a falsely elevated serum CEA level?

(A) Chronic wound infection *(B) Smoking (C) Hyperglycemia (D) Age (E) Postoperative chemotherapy

16. An 18-year-old soccer player presents to her doctor 2 days after getting kicked in the knee during her championship game. She reports hearing a snap immediately after the injury, but her knee looked normal. The next day, she had a swollen kneecap with progressive pain. On physical exam, she has medial joint line tenderness and effusion. Her doctor notices an audible snap occurring when extending the knee from a fully flexed position while applying tibial torsion. What is the most likely diagnosis?

(A) Collateral ligament tear (B) Anterior cruciate ligament tear (C) Stress fracture (D) Tendon displacement *(E) Meniscus tear

2. A 30-year-old man with colon cancer secondary to familial adenomatous polyposis (FAP) arrives for follow- up after receiving a total proctocolectomy with end ileostomy. He was found to have colon cancer after presenting at the age of 27 with unexplained rectal bleeding, diarrhea, and abdominal pain. Subsequent colonoscopy found multiple adenomatous polyps in his colon. He has a 5-year-old son, who is screened and is positive for the APC gene. What is the recommended screening for his son?

(A) Colonoscopy starting at age 20 *(B) Flexible sigmoidoscopy starting at age 10- if child tests + for APC, then flex sig starting at 10 (C) Colonoscopy starting at age 50 (D) Annual fecal occult blood test (E) Annual barium enema

8. A 63-year-old man is admitted to the hospital for alcohol pancreatitis. At 48 h after admission, he manifested four of Ranson's criteria. On hospital day 6 he is reevaluated on rounds for increasing epigastric abdominal pain. He denies any vomiting. On physical exam temperature is 100.3 °F, blood pressure is 134/74 mmHg, pulse is 89/min, and res- pirations are 16/min. The belly is distended but soft, and there is still significant epigastric tenderness. CT scan is obtained and shows diffuse edema surrounding the pan- creas with a pancreatic phlegmon, but no evidence of necrosis. What is most appropriate approach to his nutritional management?

(A) Continue NPO and intravenous normal saline (B) Clear liquid diet *(C) Enteral nutrition via feeding tube (D) Parenteral nutrition via central line (E) Parenteral nutrition via peripheral line

18. A 62-year-old female arrives to the ER with acute abdominal pain. She has a past medical history significant for diverticulitis. She is diagnosed with uncomplicated diverticulitis and managed with bowel rest and antibiotics. A year later, she has another episode of diverticulitis that was again successfully managed on an outpatient basis. Which of the following is this patient at higher risk for developing?

(A) Crohn's disease (B) Ulcerative colitis (C) Colon cancer (D) Anal cancer *(E) Stricture formation

3. A 60-year-old man presents with yellowing of his skin. He reports that he has unintentionally lost 10 lb over the last 5 months. He denies abdominal pain or fevers. He has also developed pruritus, dark urine, and clay-colored stools. He has smoked two packs per day for the past 40 years. On physical exam, his temperature is 98.6 °F, blood pressure is 110/86 mmHg, and pulse is 94/min. On physical examination, he appears jaundiced and has scleral icterus. He has fullness, suggestive of a mass in his RUQ that is not tender to palpation. What is the best term to describe this constellation of findings?

(A) Cullen's sign (B) Charcot's triad (C) Reynold's pentad *(D) Courvoisier's sign (E) Murphy's sign

12. A 90-year-old man with Alzheimer's disease arrives from a nursing home with abdominal distention and pain for the past 12 h. He takes hydrochlorothiazide, donepezil, aspirin, and docusate. His physical exam reveals a temperature of 102 °F, blood pressure 90/70 mmHg, and pulse 112/min. His abdomen is rigid and diffusely tender with rebound and guarding. Laboratory examination is signifi cant for WBC of 15 × 10 3 /μL (normal 4.1- 10.9 × 10 3 /μL), BUN of 25 mg/dL (7-21 mg/dL), and creatinine of 1.8 mg/dL (0.5-1.4 mg/dL). X-ray shows a markedly distended colon with a bent-inner tube sign. Following IV fl uids and antibiotics, what is the best next step in management?

(A) Decompression with colonoscopy (B) CT scan of the abdomen and pelvis *(C) Exploratory laparotomy (D) Placement of a rectal tube (E) Admit to hospital for close observation

5. A 17-year-old female presents with breast pain that she noticed for several months. She states that she feels multiple breast masses in both breasts. She denies any weight loss, fevers, or night sweats. She has no medical history or family history of breast cancer. Physical examination reveals that her heart has a regular rate and rhythm. The skin around the breast and areola are normal with no rashes or lesions. No solitary breast masses are palpable, but both breasts are lumpy and painful to palpation, most notably in the upper outer quadrants. There is no axillary lymphadenopathy. What is the most appropriate next step in management?

(A) Diagnostic mammography (B) Excisional biopsy (C) Ultrasound-guided core needle biopsy *(D) Reassurance and reexamine in 1 month (E) Fine-needle aspiration (FNA)

2. A 2-day-old male infant was diagnosed prenatally with Down syndrome. Delivery was uneventful, but pregnancy was complicated by polyhydramnios. The infant has had several episodes of bilious vomiting after breast feeding but is otherwise stable. Which of the following would be the most likely finding on further work-up? (A) Dilated loops of small bowel with air-fluid levels on abdominal x-ray (B) Inability to pass a nasogastric tube (C) Abdominal distention with erythema of the overly- ing skin (D) Two large air bubbles on abdominal x-ray (E) Narrowing of the distal rectum on GI contrast study

(A) Dilated loops of small bowel with air-fluid levels on abdominal x-ray (B) Inability to pass a nasogastric tube (C) Abdominal distention with erythema of the overly- ing skin *(D) Two large air bubbles on abdominal x-ray (E) Narrowing of the distal rectum on GI contrast study

8. A 76-year-old man is driven to the ED by his wife and is complaining of severe chest pain that started 30 min ago. He denies abdominal or extremity pain. Pulses in arms and legs are 2+. His kidney function is normal. CT scan shows an aortic dissection. Which of the following findings on CT scan would most strongly indicate the need for urgent surgery?

(A) Dissection of entire descending thoracic aorta (B) Involvement of common iliac arteries (C) Involvement of renal arteries (D) Extension into mesenteric vessels *(E) Involvement of origin of innominate artery

16. A 50-year-old woman undergoes screening colonoscopy. During the procedure, she is found to have a dark discoloration of the entire colon with lymph follicles shining through as pale patches. What is the most likely underlying etiology?

(A) Drinking lots of prune juice (B) Colon cancer *(C) Laxative abuse (D) Normal anatomic variation (E) Inflammatory bowel disease

18. A 34-year-old pregnant female at 36 weeks gestation undergoes ultrasound at her obstetrician's office. The ultrasound reveals polyhydramnios. Which of the following is the LEAST likely diagnosis? (A) Duodenal atresia (B) Esophageal atresia (C) Maternal diabetes (D) Fetal posterior urethral valve (E) Fetal anencephaly

(A) Duodenal atresia (B) Esophageal atresia (C) Maternal diabetes *(D) Fetal posterior urethral valve (E) Fetal anencephaly

6. A 71-year-old woman is evaluated for a lump in her right breast that she noticed 3 weeks ago. She denies any nipple discharge, nipple retraction, or skin changes. She has a sister who was diagnosed with breast cancer at the age of 57. She had menarche at the age of 9 and menopause at the age of 56. She had two children, one at the age of 39 and the other at the age of 41. On physical exam, the breasts are normal on inspection. Palpation reveals a 1.5 cm dominant lump that does not appear to be fixed to the surrounding structures in the left upper, outer quadrant. The patient has no other dominant masses in either breast. There is no axillary lymphadenopathy. What is the biggest risk factor in this patient predisposing her to breast cancer?

(A) Early menarche (B) Family history of breast cancer *(C) Older age (D) Age at first pregnancy (E) Late menopause

6. A worried mother presents to you with concerns that her 6-month-old boy has a large protrusion at his belly button that is worse when he cries but reduces when he is sleeping. On exam you palpate a 1 cm fascial defect at his umbilicus. Which of the following is true about this condition?

(A) Elective repair is recommended (B) The condition is associated with cardiac anomalies (C) The size of the defect predicts that it will not likely close on its own (D) The risk of incarceration is significant *(E) Repair should be delayed until the child is 4 years old

4. A newborn female infant is born to a 19-year-old G1P0 mother who smokes. On physical exam, the small bowel is eviscerated through an abdominal wall defect to the right of the umbilicus. The small bowel appears matted and dilated. The infant appears to otherwise be healthy. Which of the following would have been expected in prenatal screening?

(A) Elevated b-hCG levels (B) Decreased estradiol levels *(C) Elevated alpha-fetoprotein levels (D) Oligohydramnios (E) None of the above

3. A 30-year-old woman is recovering from an open cholecystectomy in the hospital. On the second postoperative day, she begins to complain of cramping abdominal pain without vomiting. She has no past medical or surgical history, and her postoperative course has been unremarkable. She is receiving oral hydrocodone for pain and is on a clear liquid diet. She has a temperature of 99.5 °F, blood pressure is 128/84 mmHg, and pulse is 82/min. Her physical exam is signifi cant for absent bowel sounds, a mildly distended abdomen with mild diffuse tenderness without rebound or guarding. Which of the following would most benefi t her abdominal fi ndings?

(A) Encouraging ambulation (B) Placement of a nasogastric tube (C) Neostigmine *(D) Conversion of hydrocodone to a nonsteroidal antiinfl ammatory drug (E) Return to the operating room for exploration

7. A 65-year-old man presents to the ED with RUQ pain. He is diagnosed with acute cholecystitis and undergoes a cholecystectomy the following day. He is discharged shortly after his procedure. Five days later, he arrives back to the ED with abdominal pain and low-grade fevers. On physical examination, his blood pressure is 120/70 mmHg, heart rate is 90/min, and temperature is 99.0 °F. He has diffuse mild abdominal tenderness to palpation. His laboratory examination is significant for white blood count of 16.9 × 103/μL (normal 4.1- 10.9 × 103/μL). What is the next best step?

(A) Endoscopic ultrasound (B) Exploratory laparotomy *(C) CT scan of abdomen (D) ERCP with stenting (E) HIDA scan

A 55-year-old man presents with a mass in the left groin that is intermittently painful. The mass protrudes upon straining and reduces when he is in the supine position. With the patient standing, there is an obvious mass in his left scrotum that protrudes from the internal ring and becomes more prominent when the patient coughs. Elective surgery is recommended. At surgery, the posterior wall of the hernia sac feels very thickened and is consistent with a possible sliding hernia. Which of the following is true regarding this type of hernia?

(A) Every attempt should be made to excise the entire sac *(B) It poses a higher risk of colon injury during repair (C) It is more common on the right side (D) It is most often associated with direct inguinal hernias (E) The hernia sac should be divided at the internal ring

18. A 17-year-old male comes to the emergency department for the third time over the last month due to pain in his right thigh above his knee that is particularly bothersome at night. He was previously diagnosed with growing pains. Now he has developed swelling above his right knee and states his pain is worse. On physical exam, there is a soft tissue mass that is tender to palpation. The skin overlying the mass is erythematous. An X-ray is obtained and shows a "sunburst" pattern in the distal femur. What is the most likely diagnosis?

(A) Ewing's sarcoma *(B) Osteosarcoma (C) Osteomyelitis (D) Osteochondroma (E) Osteoid osteoma

13. A 75-year-old male with severe aortic stenosis has a routine check-up at his primary care doctor. Which of the following symptoms portends the worst prognosis?

(A) Exertional chest pain *(B) Swollen legs (C) Fainting spells (D) Mid-systolic murmur heard loudest at the upper right sternal border (E) Small head nodding movements at each heartbeat

17. A 25-year-old male arrives to the ED with a stab wound lateral to his umbilicus after being involved in a drunken fight at a local bar. You can smell alcohol on his breath, and he is uncooperative during the exam. His temperature is 99.2 °F, blood pressure is 90/60 mmHg, and pulse is 120/min. His abdomen is soft, non-tender with no rebound or guarding. What is the most appropriate next step in management? (A) Exploratory laparotomy (B) Local wound exploration (C) CT scan of the abdomen (D) Serial physical examination (E) FAST exam

(A) Exploratory lap hemodynamically unstable w/ penetrating abdominal wound: peritonitis bowel evisceration uncooperative

17. A 88-year-old man with Parkinson's disease and COPD is brought to the ED from a skilled nursing facility because his nurse noticed bright red blood in his adult diapers. His medications include hydrochlorothiazide, metformin, levodopa, salmeterol, and docusate. On admission, he is afebrile with normal blood pressure and pulse. Nasogastric tube (NGT) aspiration returns yellow bile. Colonoscopy shows bright red blood within the colon, multiple diverticula, but due to the large amount of stool and blood clots, no active bleeding sources are able to be seen. He continues to have blood per rectum over the next hour. Blood pressure is 120/70 mmHg and heart rate is 90/min. Which of the following is the most appropriate next step in management?

(A) Exploratory laparotomy (B) Transfuse two units of packed red blood cells *(C) 99mTc red blood cell (RBC) scintigraphy (D) Esophagogastroduodenoscopy (EGD) (E) Omeprazole and antibiotics

11. A 12-year-old boy presents to the doctor for a lump in his neck. He is healthy with no previous medical prob- lems. On physical examination, he has a well-defined anterior neck mass, located in the midline and above the cricoid cartilage. The mother states that she has noted the lesion since he was about 2 years old. It does not bother him. On physical examination, the mass elevates with swallowing and is non-tender. He has no cervical adenopathy and no other complaints. The neck mass is described as a hypoechoic mass on ultrasonography. A subsequent thyroid scintogram is performed and con- firms the thyroid gland is in its correct anatomic position. Which of the following would be recommended next for this mass?

(A) FNA biopsy *(B) Proceed to surgical excision (C) Reassurance and observation (D) TSH and free T4 (E) CT scan

14. A 28-year-old man presents to the ED with an inability to move his right arm. After his initial work-up and imaging is completed, he is found to have a posterior dislocation of the right shoulder. Which of the following are most commonly associated with this type of injury?

(A) Fall on outstretched hand (B) Weight lifting (C) While walking dog, it suddenly tugs on leash *(D) Electrocution (E) Surfing

6. A 64-year-old homeless man is brought to the ED by paramedics after being found confused on the street. He has bruises on his head and arms with a few that appear to be new. He has alcohol on his breath. He is awake in the ED but unable to follow commands or answer questions. CT scan of the head without contrast demonstrates a crescent-shaped lesion. Which of the following is most associated with this patient's condition?

(A) Family member with polycystic kidney disease (B) Congenital malformation *(C) Torn bridging veins (D) Injured middle meningeal artery (E) Temporal bone fracture

9. A 32-year-old female patient arrives for follow-up for new-onset hypertension. She was started on hydrochlo- rothiazide 6 months ago. During her visit, she was found to have a blood pressure of 152/98 mmHg. She also complains of recent episodes where she experiences sudden palpitations, chest pain, diaphoresis, headache, and anxiety. Her laboratory exam demonstrates a cal- cium of 13.2 mg/dl (normal 8.5-10.2 mg/dl), PTH of 102 pg/ml (10-55 pg/ml), and an elevated plasma meta- nephrine. Which of the following would be an important additional component in the workup? (A) Fasting blood glucose (B) Prolactin level (C) MRI of the sella turcica (D) Serum calcitonin (E) Serum gastrin level

(A) Fasting blood glucose (B) Prolactin level (C) MRI of the sella turcica *(D) Serum calcitonin (E) Serum gastrin level hyperparathyrodism + pheochromocytoma + medullary thyroid cancer. MEN II

1. A 30-year-old female presents with bloody discharge from her left breast that she has noticed intermittently for the past month. She denies any palpable breast mass, weight loss, fevers, or night sweats. She has no medical history or family history of breast cancer. The skin around the breast and areola are normal with no rashes or lesions. No breast mass is palpable, and there is no axillary lymphadenopathy. Ultrasound did not reveal any masses. What is the most likely diagnosis?

(A) Fibrocystic changes *(B) Intraductal papilloma (C) Ductal carcinoma in situ (DCIS) (D) Paget's disease of the breast (E) Infiltrating ductal carcinoma

19. A 2-week-old infant delivered at 35 weeks gestation is brought to the pediatrician by his mother who reports that he has had a harsh, barking cough and makes a high- pitched whistling sound when he inhales. He has been feeding poorly but has not had a fever. The mother also says that the patient has bouts of blue discoloration around his lips, more frequently when he is lying on his back than on his stomach. Chest x-ray is normal. What is the most likely diagnosis?

(A) Foreign body aspiration (B) Asthma (C) Transient tachypnea of the newborn *(D) Tracheomalacia (E) Congenital diaphragmatic hernia

16. A 42-year-old man presents with new-onset hypertension and elevated hemoglobin (19 mg/dL) and hematocrit (58 %) levels on subsequent laboratory examination. A CT scan demonstrates bilateral adrenal masses suspicious for pheochromocytoma. His elevated hemoglobin and hematocrit are believed to be secondary to a paraneoplastic syndrome. What other tumor is classically associated with this same paraneoplastic syndrome? Pheochromocytoma and associated paraneoplastic syndrome? Ectopic EPO production.

(A) Glioblastoma multiforme *(B) Hemangioblastoma (C) Colorectal cancer (D) Wilms' tumor (E) Osteosarcoma

1. A 35-year-old female telemarketer presents with complaints of pain in her right wrist for the past 6 months. She also reports having numbness in her right hand that occurs with wrist flexion or extension. For the past month, she has been receiving corticosteroid injections after her doctor diagnosed her with carpal tunnel syndrome. However, her pain has dramatically increased over the past week and has started to affect her job performance. Her mother has rheumatoid arthritis. Her hand appears to be swollen and is tender to palpation. She is scheduled for a synovial fluid aspiration. Which of the following findings in the aspirate would indicate surgical intervention?

(A) Glucose of 90 mg/dL (B) 1.8 × 103/μL white blood cells/mcl (C) High abundance of rheumatoid factor (D) Negatively birefringent crystals *(E) Thick yellow-green fluid

7. A 75-year-old man arrives to the ED with abdominal pain. He has never experienced this before but reports having left- sided abdominal pain over the last 2 days that is relieved temporarily after defecation. He has a temperature of 101.2 °F with a blood pressure of 142/88 mmHg and pulse of 88/min. His laboratory examination is signifi cant for a WBC of 14 × 10 3 /μL (normal 4.1-10.9 × 10 3 /μL) with 10 % bands. CT imaging reveals focal sigmoidal wall thickening with significant paracolic inflammation. He was treated as an in-patient for 3 days and discharged home without complications. Which of the following is the most appropriate follow-up option?

(A) High-fiber diet alone (B) CT scan 2 weeks after resolution (C) Elective sigmoid colectomy (D) Barium enema *(E) Colonoscopy

17. A 32-year-old housekeeper comes to the physician for hip pain localized to the lateral aspect of the hip. The hip pain is interrupting her sleep. She denies any muscle weakness or numbness and tingling. The pain is not worse with physical activity. On physical exam, there is tenderness to palpation on the lateral aspect of her hip while in the lateral decubitus position. Which of the following is the most likely etiology of the hip pain?

(A) Hip osteoarthritis (B) Meralgia paresthetica *(C) Trochanteric bursitis (D) Osteonecrosis (E) Hip fracture

3. A 12-day-old male born at 33 weeks becomes lethargic and hypothermic over the course of 24 h. He is not tolerating his formula feeds, has two episodes of bilious emesis, and has three episodes of bloody diarrhea. Physical exam reveals abdominal distention, visible loops of bowel, abdominal wall erythema, and absent bowel sounds. What is the most likely diagnosis? (A) Hirschsprung's disease (B) Duodenal atresia (C) Esophageal atresia with tracheoesophageal fistula (D) Necrotizing enterocolitis (E) Meconium ileus

(A) Hirschsprung's disease (B) Duodenal atresia (C) Esophageal atresia with tracheoesophageal fistula *(D) Necrotizing enterocolitis (E) Meconium ileus

14. Which of the following is the most important risk factor for aortic dissection?

(A) History of coronary artery bypass grafting (CABG) (B) Giant cell arteritis (C) Pregnancy *(D) Hypertension (E) Bicuspid aortic valve

7. Which of the following is most consistent with an aldosterone-secreting adrenal adenoma? (A) Hyperglycemia, hirsutism, and abdominal striae (B) Hypertension and hyperkalemia (C) Hypertension and hypokalemia (D) Elevated plasma metanephrine and hypertension (E) Increased vanillylmandelic acid excretion and hypertension

(A) Hyperglycemia, hirsutism, and abdominal striae (B) Hypertension and hyperkalemia *(C) Hypertension and hypokalemia (D) Elevated plasma metanephrine and hypertension (E) Increased vanillylmandelic acid excretion and hypertension

6. An elderly nursing home patient has been bedridden for several months due to a series of debilitating strokes. Past medical history is significant for hypertension, controlled with a diuretic, and Paget's disease. Recently, the patient has been complaining of vague abdominal pain, constipation, and depressed mood. On physical examination, the patient is alert and oriented. Abdominal examination is unremarkable. Which of the following electrolyte abnormalities would most likely explanation her symptoms? (A) Hyponatremia (B) Hypernatremia (C) Hyperphosphatemia (D) Hypocalcemia (E) Hypercalcemia

(A) Hyponatremia (B) Hypernatremia (C) Hyperphosphatemia (D) Hypocalcemia *(E) Hypercalcemia

4. A young man is doing some woodworking in his back- yard when the saw blade falls off the harness and on to his hand, severing his right thumb. After raising the right arm and applying direct pressure to control bleeding, which of the following is the best recommendation to keep his digit viable for potential replantation?

(A) Immediately place the digit directly in a small bag filled with cold water (B) Immediately place the digit directly on top of ice (C) Remove any dirt from the digit first, and then place directly on top of ice (D) Remove any dirt from the digit first with a clean damp cloth, and then place digit directly in cup of cold water *(E) Remove any dirt from the digit first, wrap in a cleandamp cloth, put in a plastic bag, and then place bag directly in cup of cold water

5. A 25-year-old female lifeguard presents to her doctor to discuss a new 10 mm skin lesion that she found on her right forearm that has been growing over the last month. The lesion has a heterogeneous dark blue color, is symmetric, and has been growing vertically. What is the most likely diagnosis? (A) Impetigo (B) Melanoma (C) Nevus (D) Molluscum contagiosum (E) Squamous cell carcinoma

(A) Impetigo *(B) Melanoma (C) Nevus (<6 mm) (D) Molluscum contagiosum (E) Squamous cell carcinoma

11. Which laboratory finding is consistent with obstructive jaundice? (A) Increased urine urobilinogen (B) Increased urine conjugated bilirubin (C) Increased stool stercobilin (D) Indirect > direct hyperbilirubinemia (E) Elevation of transaminases out of proportion to alkaline phosphatase

(A) Increased urine urobilinogen *(B) Increased urine conjugated bilirubin (C) Increased stool stercobilin (D) Indirect > direct hyperbilirubinemia (E) Elevation of transaminases out of proportion to alkaline phosphatase

8. A 35-year-old patient presents for a follow-up visit for an elevated serum calcium level of 12.8 mg/dL and an elevated PTH. He is a thin man without a significant past medical history. He reports that for the past 2 weeks he has been experiencing loose stools, polydipsia, and polyuria. On physical exam he was found to have large erythematous erosions with blisters over the lower abdomen. Which tumor would best explain the patient's symptoms and rash? (A) Insulinoma (B) Prolactinoma (C) VIPoma (D) Glucagonoma (E) Adrenal adenoma

(A) Insulinoma (B) Prolactinoma (C) VIPoma *(D) Glucagonoma (E) Adrenal adenoma

22. A 45-year-old female presents to her physician com- plaining of abdominal pain. She has a history of recur- rent kidney stones and was recently discharged from the hospital after undergoing ureteroscopic laser lithotripsy. Her laboratory examination is significant for calcium of 13.6 mg/dL (normal 8.5-10.2 mg/dL) and PTH of 112 pg/mL (10-55 pg/mL). She is scheduled for operative management of her underlying condition. At surgery, all four parathyroid glands are identified. Only one appears to be abnormally enlarged and is removed. Confirmation of curative resection is best achieved via:

(A) Intraoperative ultrasound (B) Intraoperative frozen section *(C) Intraoperative PTH levels (D) Immediate postoperative serum calcium level (E) Postoperative sestamibi scan

3. A 65-year-old male undergoes a videoscopic right upper lobectomy for squamous cell lung cancer. On postoperative day one, he suddenly develops chest pain and diaphoresis. Blood pressure is 120/60 mmHg, and heart rate is 80/min. Serial highly sensitive troponin I assays demonstrate levels of 0.4, 0.3, and 0.01 ng/dl. ECG demonstrates nonspecific T wave changes with no ST segment elevation. Following the administration of oxygen, morphine, aspirin, and a beta-blocker, his symptoms resolve. What is the next step in the management?

(A) Intravenous thrombolytic therapy (B) Percutaneous coronary intervention without stenting (C) Percutaneous coronary intervention with stenting (D) Coronary artery bypass graft (CABG) *(E) Continue medical management and reevaluate as outpatient in 4-6 weeks

2. Which of the following is true regarding diffuse axonal injury?

(A) It is often associated with a lucid interval *(B) Blurring of gray-white junctions is commonly found on imaging (C) It occurs following a tensile force (D) Persistent vegetative state is rare (E) Patients often have hyperdense fluid in the ventricles, sulci, and cisterns

19. A 61-year-old patient with coronary artery disease, diabetes, and hypertension presents to the ED with dif- ficulty speaking and right-sided hemiparesis. His wife reports that he had multiple episodes a few days ago where he had difficulty speaking but they only lasted a few minutes. A CT scan of the head without contrast did not identify any hemorrhage in the brain. His systolic blood pressure is found to be 230 mmHg. What is the most appropriate next step in management? (A) Labetalol (B) Nimodipine (C) Nitroglycerin (D) Nitroprusside (E) Hydralazine

(A) Labetalol Dx: ischemic stroke. Permissive hypertension allowed to maximize cerebral perfusion pressure (CPP). Avoid Nitroglycerin or nitroprusside, which can serve as potent vasodilators in cerebtral arteries/veins, increasing ICP. Increasing ICP can compromise CPP. Nimodipine in the case of hemorrhage to avoid cerebral vasospasms Hydralazine for pregnant patients

13. A 60-year-old woman arrives to the emergency depart- ment with bloody emesis. She has a past medical history significant for hypertension and an episode of severe pancreatitis due to alcohol abuse 1 year ago and has since developed chronic pancreatitis. Her temperature is 98.6 °F, blood pressure 110/88 mmHg, and pulse of 88/ min. Esophagogastroduodenoscopy shows bleeding coming from isolated gastric varices. Which of the fol- lowing is most likely to successfully treat the bleeding? (A) Liver transplantation (B) Endoscopic banding of the varices (C) Endoscopic sclerotherapy (D) TIPS (transjugular portosystemic shunt) (E) Splenectomy

(A) Liver transplantation (B) Endoscopic banding of the varices (C) Endoscopic sclerotherapy (D) TIPS (transjugular portosystemic shunt) *(E) Splenectomy (splenic vein thrombosis + gastric vein collateral --> isolated gastric varices)

6. A 6-h-old male infant is noted to be dyspneic with an oxygen saturation of 86 %. Physical exam reveals subcostal retractions and moderate perioral cyanosis. Which finding on chest x-ray would be most suggestive of a nonsurgical diagnosis? (A) Loops of bowel in the left chest (B) Tip of orogastric tube located above carina (C) Diffuse pulmonary interstitial edema (D) Boot-shaped heart with upturned apex (E) Double-bubble sign just beneath the diaphragm

(A) Loops of bowel in the left chest (B) Tip of orogastric tube located above carina *(C) Diffuse pulmonary interstitial edema (D) Boot-shaped heart with upturned apex (E) Double-bubble sign just beneath the diaphragm

4. Which of the following findings would be expected in a patient presenting with a transtentorial (uncal) herniation?

(A) Loss of gag reflex (B) Diplopia on attempted lateral gaze (C) Medial rectus palsy on attempted lateral gaze *(D) Ptosis and a "down and out" eye (E) Paralysis of the sternocleidomastoid musc

7. A 50-year-old woman comes to clinic to discuss treat- ment for a new diagnosis of breast cancer. Her annual screening mammogram revealed a 1.3 cm mass in the right breast. The patient does not have any other breast masses, skin changes, nipple discharge, or axillary ade- nopathy. Mammography revealed no other suspicious calcifications within the breast. Biopsy of the mass was performed and revealed infiltrating ductal carcinoma. Estrogen receptor, progesterone receptor, and Her2/neu receptor testing were negative. Which of the following is the best option for the management of this patient's breast cancer?

(A) Lumpectomy and breast irradiation (B) Lumpectomy and hormone therapy (C) Lumpectomy and chemotherapy (D) Lumpectomy, sentinel node biopsy, and breast irradiation *(E) Lumpectomy, sentinel node biopsy, breast irradiation, and chemotherapy

20. An ambitious young surgeon decided to travel to Somaliland with an international medical team to provide services for patients that have not had any previous medical attention. His work involved traveling long distances by foot and standing up for many hours while performing minor and major surgeries for an intense 3-week period. After returning from the trip, he experiences fatigue and increasing pain upon palpation at the midpoint of his right leg that wakes him up at night. On physical exam, he has point tenderness in his proximal tibia with some surrounding edema of the skin. Plain X-ray of the tibia is negative. What is the most likely etiology for his pain?

(A) Medial tibial stress syndrome (MTSS) (B) Osgood-Schlatter *(C) Stress fracture (D) Exertional compartment syndrome (E) Osteomyelitis

6. A 16-year-old boy arrives for his 1 month follow-up after straining his back and fracturing his tibia during a snowboarding accident. He has a long leg cast and uses crutches to get around. He reports that he lost the crutches given to him upon discharge but has been using an old pair that belonged to his older brother. His only complaint is a weak left hand that sometimes "falls" when he stretches out his arm. On physical exam, he has sensory deficits on the lateral dorsal side of his left hand. His leg is not bothering him and appears to be healing well. What is the most likely explanation for his upper extremity abnormalities?

(A) Missed midshaft humerus fracture *(B) Radial nerve injury (C) Long thoracic nerve injury (D) Ulnar nerve injury (E) Musculocutaneous nerve injury

4. A Richter's hernia:

(A) Most often contains colon or bladder in the posterior aspect of the sac (B) Has a low risk of incarceration (C) Most commonly presents as a small bowel obstruction *(D) Can mislead the clinician as strangulated bowel can easily be missed (E) Should be manually reduced in the emergency department provided there is no evidence of bowel obstruction

3. A 24-year-old woman presents to the ED with shoulder pain after a sports injury. She is found to have an anterior dislocation of the humeral head from the glenoid fossa. Which of the following is most likely to be injured in association with this type of dislocation?

(A) Musculocutaneous nerve (B) Radial nerve *(C) Axillary nerve (D) Brachial arterial (E) Medial clavicle

5. A 65-year-old woman arrives to the ED complaining of chest pain. Her past medical history includes hypertension, atherosclerosis, and coronary artery disease. She underwent a coronary artery bypass graft (CABG) 3 weeks ago for three-vessel disease. She reports that her chest pain worsens with inspiration and lessens when leaning forward. A friction rub is heard on auscultation. ECG shows global ST elevation. What is the most likely diagnosis?

(A) Myocarditis (B) Myocardial infarction (C) Cardiac tamponade *(D) Acute pericarditis (E) Pulmonary embolism

13 A 29-year-old male with no past medical history presents to the ED with the worst headache of his life. He was watching television when the headache started and is unlike any headache he has experienced before. It is located near the back of his head. He reports that his father had a kidney transplant at a young age. He is managed appropriately for his acute condition. His recovery is complicated by progressive lethargy, agitation, and eventual coma. Which of the following would most likely explain this complication? (A) Nephrogenic diabetes insipidus (B) Central (neurogenic) diabetes insipidus (C) Poor oral intake (D) Cerebral salt-wasting syndrome (E) Thyroid dysfunction

(A) Nephrogenic diabetes insipidus (B) Central (neurogenic) diabetes insipidus (C) Poor oral intake *(D) Cerebral salt-wasting syndrome (E) Thyroid dysfunction

11. A 45-year-old female presents with a recent change in a preexisting mole on her anterior thigh. She states that the mole keeps bleeding, is darker, and has grown. The mole is 8 mm in diameter on physical exam. There are no palpable nodes in the groin. An excisional biopsy is performed with a 1 mm margin, and to a depth of the subcutaneous fat. Pathology reveals a melanoma that is 0.5 mm in thickness. The margins are negative. What is the next step in the management? (A) No further treatment (B) Re-excision with 1 cm margins (C) Interferon alpha (D) Granulocyte-macrophagecolony-stimulatingfactor (GM-CSF) (E) Dacarbazine

(A) No further treatment *(B) Re-excision with 1 cm margins (C) Interferon alpha (D) Granulocyte-macrophagecolony-stimulatingfactor (GM-CSF) (E) Dacarbazine

15. A 61-year-old man with a past medical history of opioid dependence and diabetes presents with focal back pain and right leg weakness. He does not recall any recent trauma. He reports having "back surgery" nearly 20 years ago but does not remember why. He also reports helping his friend move furniture last week. His temperature is 101.4°F, blood pressure is 104/76 mmHg, and pulse is 102/min. Physical examination is significant for focal pain on palpation of his lower lumbar spine. Laboratory examination is significant for elevated ESR. Imaging is obtained. What is the best management approach for this condition?

(A) Nonsteroidal anti-inflammatory drugs (NSAID) (B) High-dose intravenous corticosteroids (C) Long-term antibiotics alone (D) Physical therapy and education *(E) Long-term antibiotics with surgical drainage

13. A 44-year-old man presents with right lower quadrant pain without rebound, nausea, and vomiting. CT scan shows bowel wall thickening near the ileocecal valve. He is scheduled for an appendectomy. Final pathology confirms acute appendicitis. In addition, an incidental 2.5 cm carcinoid tumor is found at the tip of the appendix. What is the most appropriate next step in management?

(A) Observation (B) Colonoscopy within the next 6 months (C) UGI with small bowel follow through (D) Chest X-ray *(E) Right hemicolectomy tumor < 1 cm -> appendectomy sufficient tumor > 1 cm and located at base of appendix or > 2 cm on tip of appendix -> right hemicolectomy.

24. A 38-year-old female arrives for her yearly physical. She has no complaints but was incidentally found to have laboratory markers suggestive of primary hyper- parathyroidism. Subsequent workup reveals involve- ment of all four parathyroid glands. She remains asymptomatic. What is the best recommendation for management of this patient? (A) Observation (B) Surgical removal of all four glands (C) Surgical removal of 3.5 glands (D) Biochemical monitoring of serum calcium and serum creatinine annually (E) Cinacalcet

(A) Observation (B) Surgical removal of all four glands *(C) Surgical removal of 3.5 glands (D) Biochemical monitoring of serum calcium and serum creatinine annually (E) Cinacalcet

3. A 55-year-old otherwise healthy patient undergoes a non-contrast CT abdomen to evaluate for possible kidney stones and is incidentally noted to have a 8 cm mass in the left adrenal gland. The mass has irregular borders and high attenuation, suggesting a lipid-poor lesion, and appears to be adherent to the kidney. How should this patient be managed? (A) Observation with repeat CT scan in 3 months (B) Open adrenalectomy (C) Laparoscopic adrenalectomy (D) Radiation therapy (E) Percutaneous biopsy

(A) Observation with repeat CT scan in 3 months *(B) Open adrenalectomy (C) Laparoscopic adrenalectomy (D) Radiation therapy (E) Percutaneous biopsy

14. A 56-year-old male undergoes a Whipple procedure for pancreatic adenocarcinoma. Two days later, there is about 30 cm3 of white/opal opaque drainage emanating from the patient's drain. What is the most appropriate next step? (A) Obtain abdominal CT scan (B) Obtain abdominal ultrasound (C) Send fluid for amylase level (D) Start octreotide (E) Initiate total parenteral nutrition (TPN)

(A) Obtain abdominal CT scan (B) Obtain abdominal ultrasound *(C) Send fluid for amylase level (D) Start octreotide (E) Initiate total parenteral nutrition (TPN)

4. A 50-year-old female has been recently diagnosed with primary hyperparathyroidism. She comes in to her doctor complaining of increased bone pain in her legs. She is found to have elevated serum calcium, alkaline phosphate, and PTH. Her doctor decides to order plain films of her lower extremities. The radiographs show very thin bones with a stress fracture and bowing of both femur bones. She also has characteristic cysts with a moth-eaten appearance. What is the most likely diagnosis?

(A) Osteoporosis (B) Osteopetrosis (C) Osteomalacia *(D) Osteitis fibrosa cystica (E) Paget's disease of the bone

9. A 48-year-old woman arrives for her 6-month follow-up after undergoing breast-conserving therapy to treat stage IIA breast cancer. Her only postoperative complication has been red patches of skin on her chest following radi- ation therapy. She does not complain of any pain or pru- ritus but does report having intense headaches in the mornings when she wakes up. Her temperature is 99.1°F blood pressure is 110/80 mmHg, and pulse is 90/min. Her physical examination is normal. She is worried that the red patches on her chest are suggestive of something more serious. Which of the following studies would be the most appropriate next step in management?

(A) PET (B) Cerebral angiogram (C) Skin biopsy (D) Carotid duplex *(E) CT scan of the head

13. A 42-year-old man with a family history of endocrine tumors is diagnosed with MEN-2A after presenting with uncontrolled hypertension and subsequent genetic workup. He was found to have a right adrenal pheochromocytoma and asymptomatic hyperparathyroidism. What is the recommended surgical management for this patient? (A) Parathyroid surgery first, followed by adrenalectomy (B) Adrenalectomy first, followed by parathyroid surgery (C) Medical conditioning for 2 weeks prior to adrenalectomy, followed by parathyroid surgery (D) Medical conditioning for 2 weeks prior to simultaneous parathyroid surgery and adrenalectomy (E) Medical conditioning for 2 weeks followed by adrenalectomy only MEN2A: pheochromocytoma, medullary thyroid cancer, hyperparathyroidism

(A) Parathyroid surgery first, followed by adrenalectomy (B) Adrenalectomy first, followed by parathyroid surgery *(C) Medical conditioning for 2 weeks prior to adrenal- ectomy, followed by parathyroid surgery (D) Medical conditioning for 2 weeks prior to simulta- neous parathyroid surgery and adrenalectomy (E) Medical conditioning for 2 weeks followed by adrenalectomy only

18. A malignant pheochromocytoma is diagnosed by: (A) Pathologic identification of high mitotic rate, cellular atypia, and capsular invasion (B) Positive MIBG scan (C) Presence of metastasis at sites normally devoid of chromaffin tissue (D) Biomolecular markers (E) The presence of intractable hypertension

(A) Pathologic identification of high mitotic rate, cellular atypia, and capsular invasion (B) Positive MIBG scan *(C) Presence of metastasis at sites normally devoid of chromaffin tissue (D) Biomolecular markers (E) The presence of intractable hypertension

10. A 45-year-old Internet insurance salesman presents to his doctor with progressive pain in his left hand for 1 year. His pain is primarily limited to the left thumb and index finger. After finishing the physical examination his doctor believes he has carpal tunnel syndrome (CTS). Which of the following did the doctor most likely find on physical examination to support this diagnosis?

(A) Percussing the wrist elicits pain in all five fingers and the palmar surface *(B) Pain felt along the left thumb, index, middle finger when placing the elbow on a table and flexing the wrist for 60 s (C) Pain felt along the left thumb, index, middle finger, and palmar surface when placing the elbow on a table and flexing the wrist for 60 s (D) Radial deviation of the wrist upon wrist flexion (E) Posterior and lateral forearm sensory deficit

21. During the course of a total thyroidectomy in a 40-year- old female, the surgeon divides the superior thyroid artery and vein in one large ligature. After dividing the vascular pedicle, the surgeon notices that it appears that a nerve was transected. The surgeon postoperatively should warn the patient that she will most likely have: (A) Permanent hoarseness (B) A droop in the corner of her mouth (C) Difficulty swallowing (D) Trouble hitting high notes when singing (E) A need for a permanent tracheostomy

(A) Permanent hoarseness (B) A droop in the corner of her mouth (C) Difficulty swallowing *(D) Trouble hitting high notes when singing (E) A need for a permanent tracheostomy

23. A 35-year-old female presents with bone pain, abdominal pain, and depressed mood. Her laboratory examination is significant for calcium of 11.3 mg/dL (normal 8.5-10.2 mg/ dL) and PTH of 109 pg/ml (10-55 pg/mL). Localization of the enlarged gland or glands is best achieved by:

(A) Preoperative MRI (B) Preoperative ultrasound *(C) Preoperative sestamibi scan (D) Preoperative FNA (E) Intraoperative exploration of all four glands

1. A 65-year-old male presents with a painful nodule in his wrist that is determined to be a ganglion cyst. Despite attempts at aspiration, it recurs. He is unable to work as a computer programmer, is on disability, and is feeling depressed. He is scheduled for wrist surgery. He reports having been discharged 1 week ago for an episode of chest pain. Troponins were elevated at that time, but there was no elevation of his ST segment. Which of the following is the best recommendation?

(A) Proceed with surgery with intraoperative trans esophageal echocardiography (B) Proceed with surgery but perform under local anesthesia with sedation (C) Proceed with surgery only if echocardiogram shows normal ejection fraction (D) Proceed with surgery after aggressive beta blockade to get heart rate into low 60s *(E) Postpone surgery for at least 4 weeks

5. A 45-year-old healthy woman arrives for follow-up after her primary care physician discovered gallstones inci- dentally while performing imaging studies for an unre- lated event. She has no complaints and has a healthy diet but is worried about the stones. An abdominal ultra- sound is repeated and demonstrates several stones in her gallbladder without any wall thickening. What is the recommended management for this patient?

(A) Prophylactic cholecystectomy (B) Ursodeoxycholic acid (C) Endoscopic retrograde cholangiography (ERCP) *(D) Observation (E) Extracorporeal shock wave lithotripsy

15. A patient is diagnosed with type A aortic dissection, and there is concern for cardiac tamponade. Which of the following findings would be the MOST consistent with cardiac tamponade?

(A) Pulsus bisferiens (B) Watson's water hammer pulse (C) Peaked T waves *(D) Equalization of central pressures (E) Pulsus alternans

14. A 31-year-old male with HIV presents with headaches and night sweats. He is poorly compliant with his medications. His headaches are severe in the morning but subside throughout the day. He has no other complaints. His last CD-4 count was 95/mm3. His temperature is 98.9°F, blood pressure is 110/76 mmHg, and pulse is 88/min. His physi- cal exam is normal. CT scan of the head shows a ring- enhancing lesion near his right primary motor cortex. What is the best next step in management?

(A) Radiation (B) Chemotherapy and radiation *(C) Pyrimethamine and sulfadiazine (D) Stereotactic brain biopsy (E) Restart highly active antiretroviral therapy(HAART)

16. A 20-year-old male presents with a right testicular mass. He noticed the mass after getting kneed in the groin during a basketball game 3 weeks earlier. He states that the mass is not tender. He denies any other symptoms. On physical examination, there is a firm 2-cm mass within the right testicle that is not tender. Ultrasound confirms a 2-cm homogeneous mass within the right testicle. CT scan of the abdomen and pelvis is negative. CXR is negative. Which of the following is the most appropriate next step in management? (A) Radical inguinal orchiectomy (B) Scrotal orchiectomy (C) Fine needle aspiration (D) Open testicular biopsy (E) Percutaneous biopsy

(A) Radical inguinal orchiectomy inguinal incision allows longer spermatic cord removal

12. A 45-year-old woman presents with pain in her right hand for 2 days. She reports falling down and attempting to break her fall with an outstretched hand. Physical examination is significant only for pain in the anatomic snuffbox of her right hand. She has no motor weakness or sensory deficits. What is the most appropriate next step in management?

(A) Radiograph of hand and, if normal, recommend supportive care with follow-up *(B) Radiograph of hand and, if normal, recommend thumb spica cast with follow-up (C) Thumb spica cast without radiograph (D) Supportive care without radiograph (E) CT scan of hand

19. A 58-year-old female underwent a CT scan of her abdomen and pelvis following a motor vehicle collision 1 month ago. She has no intra-abdominal injuries and was discharged from the ED. However, her gallbladder was incidentally noted to be heavily calcified. She is otherwise in good health and denies any abdominal pain. Her past history is significant for mild hypertension. Which of the following is the best recommendation? (A) Reassure patient that no follow-up is needed (B) Repeat CT scan in 1 year (C) Laparoscopic cholecystectomy (D) Check serum calcium and PTH levels (E) Obtain ERCP with brushings

(A) Reassure patient that no follow-up is needed (B) Repeat CT scan in 1 year *(C) Laparoscopic cholecystectomy (D) Check serum calcium and PTH levels (E) Obtain ERCP with brushings

11. A 2-week-old female born at 39 weeks gestation to a 36-year-old G2P1 female via normal spontaneous vaginal delivery comes to her pediatrician for her 2-week well-baby exam. The birth was unremarkable, and the baby went home with her mother after 2 days. She received prenatal care starting at 10 weeks gestation and had a normal, healthy pregnancy with no issues. On physical exam, as the hip is gently adducted and posterior pressure is applied, there is a palpable clunk. Which of the following is the next step in management?

(A) Reexamine at 1-month exam (B) CT scan of the hips (C) X-ray of the hips (D) Reassurance *(E) Ultrasound of the hips

3. A 64-year-old man who emigrated from Japan arrives to his doctor to discuss new skin lesions. His wife first noticed two discolored plaques on his back 2 weeks ago, but he now has multiple lesions all over his back, chest, and face. They are the size of a coin and appear to have a "stuck-on" appearance. He is afebrile, blood pressure is 136/86 mmHg, and he has a pulse of 90/min. The skin lesions do not itch, and they are not tender. He has no other complaints, and a review of systems is negative. He is more concerned about his cosmetic appearance. What is the best next step in management? (A) Reexamine in 2 weeks (B) Skin biopsy (C) Abdominal CT scan (D) Mohs procedure (E) Corticosteroids

(A) Reexamine in 2 weeks (B) Skin biopsy *(C) Abdominal CT scan (D) Mohs procedure (E) Corticosteroids

5. A 60-year-old man is found to have a 3 cm right adrenal mass on CT scan which was obtained a month earlier following a MVC. He is asymptomatic, and does not report a history of hypertension or diabetes. What is the most appropriate next step in management?

(A) Repeat CT scan in 6 months (B) Percutaneous needle biopsy *(C) Biochemical workup for hormone excess (D) Laparoscopic adrenalectomy (E) No further follow-up is necessary

17. A 40-year-old male presents with acute chest pain and nausea. Serum troponin levels are elevated, and the ECG demonstrates ST segment elevation. Which of the following would be the strongest contraindication to intravenous thrombolytic therapy?

(A) Right knee arthroscopic surgery 1 month ago (B) Recently completed antibiotic course for H. pylori infection *(C) Wide mediastinum on CXR (D) History of alcohol abuse (E) Endovascular aortic aneurysm repair 1 month ago

16. A 45-year-old presents with a 1 day history of RUQ pain and tenderness, nausea, and vomiting. Physical examina- tion is significant for marked RUQ tenderness and guard- ing. Laboratory values are significant for a WBC of 12 × 103/μL (normal 4.1-10.9 × 103/μL) with 10 % bands, total bilirubin of 1.2 mg/dL (0.1-1.2 mg/dL), AST of 110 μ/L (normal 5-35 μ/L), ALT of 120 μ/L (7-56 μ/L), and alkaline phosphatase of 90 μ/L (33-131 μ/L). RUQ ultrasound reveals several gallstones, a thickened gallbladder wall, and a normal common bile duct. Optimal management consists of: (A) Schedule for elective outpatient laparoscopic cholecystectomy (B) Admit, IV antibiotics, laparoscopic cholecystectomy within 48 h of admission (C) Admit, IV antibiotics for 4-5 days followed by laparoscopic cholecystectomy (D) Admit, IV antibiotics until WBC normalizes, fol- lowed by outpatient laparoscopic cholecystectomy (E) Admit, IV antibiotics, ERCP, followed by laparoscopic cholecystectomy

(A) Schedule for elective outpatient laparoscopic cholecystectomy *(B) Admit, IV antibiotics, laparoscopic cholecystectomy within 48 h of admission (C) Admit, IV antibiotics for 4-5 days followed by laparoscopic cholecystectomy (D) Admit, IV antibiotics until WBC normalizes, fol- lowed by outpatient laparoscopic cholecystectomy (E) Admit, IV antibiotics, ERCP, followed by laparoscopic cholecystectomy

14. A 2-h-old male born at 39 weeks gestation with difficulty breathing is diagnosed with esophageal atresia with a tracheoesophageal fistula and undergoes surgical repair, which was uncomplicated. Ten years later, the patient develops difficulty swallowing and often vomits undigested food shortly after eating. Which of the following is the most likely explanation for this? (A) Scleroderma (B) Gastroesophageal reflux (C) Esophageal cancer (D) Esophageal leak (E) Esophageal stricture

(A) Scleroderma (B) Gastroesophageal reflux (C) Esophageal cancer (D) Esophageal leak *(E) Esophageal stricture

4. A 27-year-old man arrives to the emergency department complaining of bloody diarrhea and rectal urgency. He reports a normal appetite, and has not lost any signifi - cant weight. After initial workup yields no fi ndings, he is referred to a gastroenterologist for a colonoscopy. He is found to have pseudopolyps in his colon, and subsequent biopsy results confi rm ulcerative colitis. He is started on corticosteroids and sulfasalazine, which is able to control his symptoms. Which of the following is true regarding colon cancer and screening in patients with ulcerative colitis?

(A) Screening for colon cancer is not necessary *(B) Screening colonoscopy with random biopsies 8 years after disease onset (C) Screening colonoscopy with biopsy only if a suspicious polyp is seen (D) Screening colonoscopy annually once diagnosis is established (E) The risk of colon cancer is much less than with Crohn's disease

15. Which of the following is an appropriate use of CA 19-9? (A) Screening normal, healthy patients for pancreatic cancer (B) Screening at-risk patients for pancreatic cancer (C) Confirming diagnosis of pancreatic cancer in patients with periampullary mass on CT (D) Monitor for progression of disease following resec- tion and/or adjuvant therapy (E) None of the above

(A) Screening normal, healthy patients for pancreatic cancer (B) Screening at-risk patients for pancreatic cancer (C) Confirming diagnosis of pancreatic cancer in patients with periampullary mass on CT *(D) Monitor for progression of disease following resection and/or adjuvant therapy (E) None of the above

8. A 5-year-old boy is brought to the pediatrician because of left knee pain that is causing him to limp. His PMH is significant for severe asthma. He was recently hospitalized for a severe exacerbation requiring a prolonged steroid taper. On physical exam, his temperature is 98.7°F, blood pressure is 88/56 mm Hg, pulse is 76/min, and respirations are 16/min. He has a normal knee exam. Labs are drawn and show WBC of 7.4 × 103/μL (normal 4.1-10.9 × 103/μL) and Hgb of 12.7 g/dL (12-15 g/dL). ESR and CRP are within normal limits. Which of the following is the most likely diagnosis?

(A) Septic arthritis (B) Osteomyelitis (C) Slipped capital femoral epiphysis (D) Torn ACL *(E) Osteonecrosis of the hip

2. A 30-year-old female of Scottish descent presents witha nodule on her face near the corner of her eye. The lesion measures 12 mm in diameter. The borders are irregular, and the center of the lesion is dark. Which of the following is the best recommendation? (A) Shave biopsy (B) Punch biopsy (C) Excisional biopsy with 1 mm margin (D) Excisional biopsy with 5 mm margin (E) Reexamination in 2 months

(A) Shave biopsy *(B) Punch biopsy (C) Excisional biopsy with 1 mm margin (D) Excisional biopsy with 5 mm margin (E) Reexamination in 2 months

16. A 40-year-old female presents to the doctor with progressive weakness in both her arms for the past year. She is healthy and does not take any medications but reports being hospitalized 5 years ago for 2 days following a MVC in which she sustained severe and sudden whiplash injury. On physical examination, she has atrophy of the intrinsic muscles of both hands and an inability to differentiate between hot and cold in bilateral upper extremities. All other sensations are intact. What is the most likely underlying etiology?

(A) Spinal spondylosis (B) Brachial plexus injury *(C) Syringomyelia (D) Autoimmune condition (E) Herniated disk

17. A male infant is born via normal spontaneous vaginal delivery at 39 weeks gestation to a 32-year-old G1P1. She had good prenatal care. The infant is heard grunting while trying to breathe, and he shows bilateral subcostal and intercostal retractions. The patient is observed closely for several hours following delivery, and his respiratory status improves over that time. Which of the following is the most likely explanation of this patient's course? (A) Spontaneous reduction of congenital diaphragmatic hernia (B) Closure of the ductus arteriosus (C) Resorption of excess pulmonary fluid (D) Paramyxovirus infection (E) Closure of the foramen ovale

(A) Spontaneous reduction of congenital diaphragmatic hernia (B) Closure of the ductus arteriosus *(C) Resorption of excess pulmonary fluid (D) Paramyxovirus infection (E) Closure of the foramen ovale

4. A 46-year-old man is admitted to the hospital for severe epigastric pain of 12-h duration, nausea, two episodes of vomiting, and anorexia. His past medical history is significant for alcoholism and several admissions for alcohol withdrawal. On physical exam temperature is 99.6 °F, blood pressure is 137/84 mmHg, pulse is 99/ min, and respirations are 16/min. There is moderate tenderness in the epigastrium to palpation, but the abdomen is soft and no masses are felt. There is no scleral icterus and no jaundice of the skin. Laboratory examination is shown below. What is the next step in management? AST: 123 μ/L (normal 5-35 μ/L) ALT: 99 μ/L (7-56 μ/L) TBilli: 0.7 mg/dL (0.1-1.2 mg/dL) Lipase: 709 μ/L (7-60 μ/L) Alkaline phosphatase: 709 μ/L (33-131 μ/L) WBC: 11 × 103/μL (normal 4.1-10.9 × 103/μL) Hgb: 12.9 mg/dL (13.2-16.2 mg/dL)

(A) Start intravenous antibiotics (B) CT scan (C) ERCP with sphincterotomy *(D) NPO, IV hydration, and analgesics (E) Chlordiazepoxide (Librium) for alcohol withdrawal

19. Endocarditis secondary to which of the following organisms is associated with colon cancer?

(A) Streptococcus bovis (B) Clostridia septicum *(C) Steptococcus bovis and Clostridia septicum (D) Diphyllobothrium latum (E) Cryptococcus neoformans

9. Which of the following melanomas do not follow the ABCDE mnemonic? (A) Superficial spreading (B) Nodular (C) Amelanotic (D) Acral lentiginous (E) Amelanotic and nodular

(A) Superficial spreading (B) Nodular (C) Amelanotic (D) Acral lentiginous *(E) Amelanotic and nodular

8. Which of the following melanomas have the worst prognosis? (A) Superficial spreading (B) Nodular (C) Lentigo maligna (D) Acral lentiginous (E) Subungual

(A) Superficial spreading *(B) Nodular "berry-like lesion mostly symmetric, elevated, and one color, absence of radial growth phase) (C) Lentigo maligna (D) Acral lentiginous (E) Subungual

4. Which of the following would be best suited for Mohs surgery? (A) Superficial spreading melanoma in the arm (B) Nodular melanoma on the back (C) Basal cell carcinoma on the face (D) Subungual melanoma (E) Squamous cell carcinoma of the neck

(A) Superficial spreading melanoma in the arm (B) Nodular melanoma on the back *(C) Basal cell carcinoma on the face (cosmetically sensative) (D) Subungual melanoma (E) Squamous cell carcinoma of the neck

8. A 5-week-old boy presents with a 6-day history of vomiting. The parents report that his vomiting is more forceful than his usual spit ups and contain significantly more volume. The mother describes the vomit as partially digested milk. Between episodes of vomiting, the baby feeds vigorously. The parents report that he has made only one wet diaper today. On physical exam, the baby appears lethargic. Mucous membranes are dry and the anterior fontanelle is open and sunken. Capillary refill is 2 s. The growth chart reveals a 0.5-lb weight loss since his clinic visit 7 days ago. What is the next step in management? (A) Surgical intervention (B) CT of the abdomen (C) Upper GI contrast study (D) Fluid resuscitation (E) Ultrasound

(A) Surgical intervention (B) CT of the abdomen (C) Upper GI contrast study *(D) Fluid resuscitation (E) Ultrasound

9. A 65-year- old female is diagnosed with aortic dissection beginning 2 cm distal to the left subclavian artery and extending distally. Her blood pressure is 180/70 mmHg, and her heart rate is 88/min. Peripheral pulses are all 2+, and her abdomen is soft and non-tender. What is the next best step in treatment?

(A) Surgical repair (B) Aggressive IV fluids *(C) Labetalol drip (D) Endovascular repair (E) Nicardipine drip

4. A 17-year-old AfricanAmerican male presents for a pre- participation physical before track season. A harsh systolic murmur is heard at the second right intercostal space. He denies ever experiencing chest pain, dizziness, or difficulty breathing. Which of the following would be expected on further workup?

(A) T wave inversion on ECG (B) Laterally displaced PMI on palpation (C) Weak femoral pulses compared to brachial pulses *(D) Increased intensity of the murmur with Valsalva maneuver (E) Increased intensity of the murmur with squatting

9. A 57-year-old woman comes to clinic to discuss surgical treatment for a new diagnosis of breast cancer. Her annual screening mammogram revealed a 1.7 cm mass in the right breast. Biopsy of the mass was performed and revealed infiltrating ductal carcinoma. Estrogen receptor and progesterone receptor testing were negative, while HER-2 receptor testing was positive. In addition to lumpectomy and breast irradiation, the treating doctor decides to add hormonal therapy with trastuzumab to the treating regimen. What study must be done prior to start- ing trastuzumab?

(A) TSH and free T4 (B) Liver function tests *(C) Echocardiogram (D) Creatinine clearance (E) CXR Trastuzumab = high risk for cardiomyopathy

17. Which of the following is true regarding paragangliomas (extra-adrenal pheochromocytomas)? (A) The most common location is within the kidney (B) There is a decreased association with familial syn- dromes (e.g., MEN-2, Von Hippel-Lindau) compared to pheochromocytomas (C) They are less likely to be malignant compared to pheochromocytomas (D) Functional imaging (MIBG) is particularly useful to diagnose metastatic disease, particularly when CT/MRI are negative (E) They are different on a cellular level from intra- adrenal pheochromocytomas

(A) The most common location is within the kidney (B) There is a decreased association with familial syn- dromes (e.g., MEN-2, Von Hippel-Lindau) compared to pheochromocytomas (C) They are less likely to be malignant compared to pheochromocytomas *(D) Functional imaging (MIBG) is particularly useful to diagnose metastatic disease, particularly when CT/MRI are negative (E) They are different on a cellular level from intra- adrenal pheochromocytomas

2. A 66-year-old woman presents to her family doctor complaining of a pain in her left groin that has appeared intermittently over the past several months. On physical exam, a soft mass is palpated in her left groin, below the inguinal ligament, and near her femoral pulse. On palpation, the mass is soft and slightly tender and disappears with gentle compression. Which of the following is true regarding these types of hernias?

(A) They are the most common hernia type in women (B) The risk of strangulation is relatively low (C) The hernia sac travels lateral to the femoral vein (D) If discovered incidentally and the patient is asymptomatic, repair is not indicated *(E) It is associated with multigravida

5. A 4-month-old baby girl is seen at her pediatrician's office for her well-child check. The parents raise a concern that she has been vomiting approximately 1/3 of her meals since 2 weeks of age. The emesis is the color of milk and is not bile-stained. There has been no change in the frequency or amount of emesis. She is exclusively breast-fed. On physical exam, mucous membranes are moist and the anterior fontanelle is open and flat. Her growth is at the 75th percentile for height and weight and has not changed significantly since birth. She is otherwise asymptomatic and without findings on physical examination. Which of the following is the most likely diagnosis?

(A) Tracheoesophageal fistula (B) Duodenal atresia (C) Pyloric stenosis *(D) Gastroesophageal reflux (E) Malrotation

12. A 50-year-old woman arrives to the ED after she was witnessed having two generalized tonic-clonic seizures over 15 min without recovering consciousness between episodes. Her seizures are resolved by the time she gets to the hospital. She had a toe removed for a skin tumor 10 years ago but does not remember any details. Review of systems reveals a loss of nearly 15 lb over the last 2 months. She has no other complaints. CT scan of the head reveals a solitary mass in the gray-white junction of the left temporal lobe with some surrounding edema. After further imaging and work-up is done, no other lesions are identified. Which of the following most likely represents the mass?

(A) Type IV astrocytoma *(B) Metastasis (C) Oligodendroglioma (D) Meningioma (E) Ependymoma

6. An obese 52-year-old man with a 50-pack-year smoking history and hypertension controlled with chlorthalidone presents to a remote hospital without interventional capabilities with 30 min of crushing chest pain radiating to his left arm and jaw. Troponin and CK-MB levels are elevated, and ECG shows ST segment elevations in leads V1 through V4. He is treated with thrombolytic therapy, and his symptoms resolve. The next morning, the patient is found dead in his bed. Which of the following is the most likely cause of death?

(A) Ventricular free wall rupture (B) Embolic stroke *(C) Ventricular arrhythmia (D) Post-MI pericarditis (E) Overwhelming infection

14. A 39-year-old man is recovering from bilateral adrenalectomy for a pheochromocytoma. On his second postoperative day, he begins to complain of nausea, vomiting, weakness, blurry vision, and mild abdominal pain. His temperature is 102.9 °F, and blood pressure is 90/68 mmHg. His ECG shows sinus tachycardia. His laboratory examination from that morning showed: Sodium: 134 mEq/L (137-145 mEq/L) Potassium: 5.8 mEq/L (3.6-5.0 mEq/L) Calcium: 7.4 mg/dL (8.9-10.4 mg/dL) BUN: 12 mg/dL (7-21 mg/dL) Creatinine: 1.2 mg/dL (0.5-1.4 mg/dL) Glucose: 70 mg/dL (65-110 mg/dL) Albumin: 2.4 g/dL (3.5-4.8 g/dL) WBC 10.5 × 103/μL (4.1-10.9 × 103/μL) Which of the following can best explain this patient's current presentation?

(A) Volume depletion (B) Sepsis (C) Hypocalcemia *(D) Low cortisol (E) Loss of catecholamine production

6. A 65-year-old obese male with diabetes and a history of IV drug abuse presents with a painful swollen left leg. Exam reveals dark purple discoloration and several large bullae over the calf. Vitals are temperature of 101.1 °F, heart rate of 120/min, and blood pressure of 92/68 mmHg. The CRP is 200 mg/L (normal < 10 mg/L), and the WBC is 28.3/mm3 (normal 4-10/mm3). Creatinine is 2.0 mg/dL (normal 0.5-1.5 mg/dL), and Na is 127 mEq/L (normal 135-145 mEq/L). Distal pedal pulses are 1+. IV fluids and intravenous antibiotics are administered. Which of the following is the next best step? (A) X-ray of the leg (B) CT scan of the leg with IV contrast (C) Venous Duplex scan of the left leg (D) Measure compartment pressures (E) Emergent wide surgical debridement

(A) X-ray of the leg (only if equivocal) (B) CT scan of the leg with IV contrast (C) Venous Duplex scan of the left leg (D) Measure compartment pressures *(E) Emergent wide surgical debridement

15. A 65-year-old male presents with iron deficiency anemia. Fecal occult blood testing is positive. Upper endoscopy demonstrates an irregular ulcer in the body of the stomach. Biopsy confirms adenocarcinoma. Simultaneous endoscopic ultrasound (EUS) demonstrates that the mass has invaded into the submucosa. No enlarged nodes are seen around the lesion. Which of the following would be the next best step in the management? (A) Proceed to gastric resection (B) CT scan of the abdomen (C) PET scan (D) Laparoscopy (E) Chemotherapy

(B) CT scan of abdomen EUS provides accurate assessment of tumor size, depth, and lymph node involvement (therefore done first) Then, do CT for distant metasis

7. A 50-year-old female presents with massive UGI bleeding. In the ED, she is actively vomiting large amounts of bright red blood. She appears to be lethargic and pale. Blood pressure is 70/50 mmHg, heart rate is 120/min, and RR is 22/min. The next step in the management is: (A) Emergent upper endoscopy (B) Immediate orotracheal intubation (C) Place bed in reverse Trendelenburg position (D) Give 2 liter bolus of normal saline (E) Give two units of O-negative blood

(B) Intubation Intubation -> NS -> Blood -> Emergent endoscopy for cause of bleeding -> foley catheter for volume status

16. Successful eradication of H. pylori is best documented by: (A) EGD with biopsy (B) Urea breath test (C) Blood antibody test (D) Stool antigen (E) Documentation is unnecessary provided symptoms have resolved

(B) Urea breath test other options: fecal antigen test, upper endoscopy performed 4 weeks after more after therapy.

5. A young man from Armenia arrives to the ED complaining of a progressively bloody cough. He reports night sweats and fevers for the past month. He has been in the United States for 1 week visiting family. As you begin examining him, he coughs up a massive amount (200 ml) of bright red, foamy sputum and has difficulty speaking. Portable chest x-ray shows multifocal patchy and cavitary opacities in the right upper lobe with mediastinal lymphadenopathy. Given his increased risk for asphyxiation, the patient is intubated and placed in a right lateral decubitus position. What is the next best step in management? (A) Emergent thoracotomy in the OR (B) Bronchoscopy (C) Video-assisted thoracic surgery (VATS) (D) Emergency arteriography (E) INH, rifampin, ethambutol, and pyrazinamide

(B) bronchoscopy dx: TB (Armenia, Mexico, Nigeria) and CXR findings

11. A 55-year-old man starts a new security job that requires a physical exam by a doctor. He has no past medical history, and everyone in his family is healthy. He has a well-balanced diet and exercises every day. He is evaluated during a routine examination which includes a discussion of health maintenance issues. He is surprised to hear that it is recommended for a man of his age to get a colonoscopy. He asks if there are any other options available. Which of the following is an appropriate recommendation with respect to this health maintenance screening strategy? (A) Fecal occult blood test (FOBT) every 5 years (B) Annual rectal examination (C) Flexible sigmoidoscopy every 5 years + FOBT every 3 years (D) Annual CT colonography (E) Annual CEA levels

(C) Flexible sigmoidoscopy every 5 years + FOBT every 3 years

9. A 37-year-old obese woman arrives to the ED with left flank pain and hematuria. She has never experienced these symptoms before. Her past medical history includes Crohn's disease which has been controlled with mesalamine. She is afebrile with a blood pressure of 130/84 mmHg and a pulse of 104/min. Physical examination reveals a laparotomy scar in her right lower quadrant. She is given analgesics for pain control. What is the most likely etiology of her acute symptoms? (A) Gallstones (B) Hypercalciuria (C) Increased absorption of oxalate (D) Protease producing bacteria (E) Mesalamine

(C) Increased absorption of oxalate: Crohn's patients w/ ileocolic resection have bad fat absorption. Usually, the fat is absorbed at the distal ileum. Now, the undigested fat binds to calcium, leaving oxalate free to be absorbed by the GI system. If not Crohn's, then hypercalciuria most likely!

13. A 50-year-old Asian female complains of abdominal discomfort and weight loss. She has an EGD performed, and biopsy of a stomach lesion returns as mucosa-associated lymphoid tissue (MALT) lymphoma. Which of the following is the best treatment plan for this patient? (A) Chemotherapy (B) Radiation therapy (C) Combined chemotherapy and radiation (D) Clarithromycin, amoxicillin, and a proton pump inhibitor (E) Gastric resection

(D) Clarithromycin, amoxicillin, and a proton pump inhibitor

8. A 48-year-old male presents with vomiting of bright red blood. He states that he has had a 1 month history of epigastric pain that is relieved by eating food. He denies weight loss. He does not take any medications or drink alcohol. He is hemodynamically stable in the emergency department. Following resuscitation, the patient undergoes upper endoscopy which reveals a posterior ulcer in the proximal duodenum which is actively bleeding. Despite numerous attempts at cauterization and injection with epinephrine, the gastroenterologist reports that she cannot stop the bleeding and the blood pressure drops to 70/50 mmHg. The next step in the management is: (A) Angiographic embolization (B) Transfer to ICU for ongoing transfusion of blood (C) Administer vasopressin (D) Exploratory laparotomy (E) Administer octreotide

(D) Exploratory laparotomy mainstay of therapy for UGI bleeding is endoscopic intervention!!!!! (inject epinephrine, sclerosing agents, clips, cauterization)

15. A 2-week-old male infant born at 26 weeks gestation is in the neonatal ICU when he becomes hypotensive and begins passing bloody stools. Which of the following is the most likely finding? (A) Palpable olive-shaped mass in the epigastric region (B) Double-bubble sign on abdominal x-ray (C) Loops of intestine in the left hemithorax on chest x-ray (D) Gas in the walls of the intestine (E) Donut sign on abdominal ultrasound

(D) Gas in the walls of the intestine

7. A 25-year-old football player presents to the ED after sustaining a devastating tackle and hyperextension of his right knee. The knee appears to be posteriorly dislocated and the leg is swollen. Pedal pulses on the right appear to be diminished but present, whereas they are normal on the left. The remainder of his exam does not reveal any obvious signs of bleeding. What is the appropriate next step in management? (A) Fasciotomy of all four compartments of the lower leg (B) CT angiography (C) Immediate heparinization (D) Plain film of the knee, followed by reduction of the dislocation (E) MRI of the knee

(D) Plain film of the knee, followed by reduction of the dislocation

4. A 65-year-old female presents with coffee ground emesis × 3. She has had vague upper abdominal pain for the past 2 months, relieved by taking antacids. In addition, she reports an involuntary weight loss of 10 lb. She denies NSAID or alcohol use. On physical examination, her blood pressure is 110/70 mmHg and HR is 80/min. Abdominal examination reveals mild epigastric tenderness with no rebound or guarding. Laboratory values are significant for a Hgb/Hct of 8.3 g/dL (normal 12-15 g/dL) and 24 % (36-44 %) with an MCV of 80 fL (80-100 fL), total bilirubin of 3.0 mg/dL (0.1-1.2 mg/dl), alkaline phosphatase of 250 IU/L (33-131 IU/L), a GGT of 270 IU/L (6-37 IU/L), an ALT of 300 IU/L (<35 IU/L), and an AST of 320 IU/L (<35 IU/L). The most likely diagnosis is: (A) Duodenal ulcer (B) Gastric ulcer (C) Pancreatic cancer (D) Ampullary cancer (E) Dieulafoy's lesion

(D) ampullary cancer fyi, dieulafoy's lesion is massive UGI lbleeding

14. Which of the following is the first step in the treatment of a patient with Boerhaave syndrome? (A) Esophagectomy (B) Gastric decompression (C) Drainage of the mediastinum and pleura (D) Primary repair of the perforation with advancement flap coverage (E) Fluid resuscitation and IV antibiotics

(E) Fluid resuscitation and IV antibiotics

18. A 71-year-old man comes in to the physician, along with his wife, and reports a 4-month history of increasing left calf pain with exercise. He works as a grocery store manager and describes having the pain every morning after he walks from his car in the parking lot to the grocery store. The pain disappears once he is able to sit down. He has smoked one pack per day for the past 50 years. His pulse is 88/min, respirations are 18/min, and blood pressure is 156/96 mmHg. His total cholesterol is 300 mg/dL. His right ABI is 0.7. His wife is very worried and asks the physician what his prognosis is. In patients with this condition, the 5-year survival is: (A) The same as that of age- and gender-matched controls without claudication (B) Reduced primarily due to the risk of limb gangrene (C) Reduced primarily due to lung cancer (D) Reduced primarily due to coronary artery disease (E) Reduced primarily due to stroke

(E) Reduced primarily due to CAD

11. An infant is found to have an undescended left testicle shortly after birth. During his 6-month checkup, the left testicle remains undescended. What can be done to decrease this patient's risk of developing testicular cancer later in life to match that of a boy born without cryptorchidism? (A) Immediate orchiopexy (B) Orchiopexy within the next 6 months (C) Hormonal therapy (D) Observation until age two and then orchiopexy (E) Risk cannot be decreased to that of a boy without cryptorchidism

(E) risk cannot be reduced. how sad...

12. Which of the following is the most common precancer- ous skin lesion? (A) Actinic keratosis (B) Seborrheic dermatitis (C) Seborrheic keratosis (D) Compound nevi (E) Keratoacanthoma

*(A) Actinic keratosis (B) Seborrheic dermatitis (C) Seborrheic keratosis (D) Compound nevi (E) Keratoacanthoma

11. A 66-year-old man is recovering in the ICU after receiving a CABG for coronary artery disease. On the fourth postoperative day, he complains of chest pain. He is sweating, anxious, short of breath, and nauseated. ECG shows evidence of right-sided MI. His blood pressure is 98/65 mmHg. What is the next best step in management?

*(A) Administer 1 L of normal saline (B) Nitroglycerin (C) Nitroprusside (D) Nifedipine (E) Lisinopril

10. A 41-year-old man with alcoholism is admitted to the ICU with a diagnosis of acute pancreatitis. He has three Ranson's criteria on admission and two more at 48 h. He requires aggressive fluid resuscitation to maintain his blood pressure in the first 24 h, but over the next 3 days, his blood pressure stabilizes. On the third day of admission, he develops tachypnea, tachycardia, and hypoxia with oxygen saturation to 89 %. Central venous pressure is 8 mmHg. The patient is placed on nasal cannula, but the oxygen saturation remains the same. His temperature is 98.9 °F, pulse is 104/min, and blood pressure is 129/73 mmHg. A chest X-ray is obtained and shows bilateral infiltrates. Labs are drawn and shown below. What is the most likely diagnosis? AST: 75 μ/L (normal 5-35 μ/L) ALT: 92 μ/L (7-56 μ/L) WBC: 11 × 103/μL (normal 4.1-10.9 × 103/μL) Arterial blood gas: pH 7.44, PaO2 66 mmHg, PaCO2 36 mmHg

*(A) Adult respiratory distress syndrome (ARDS) (B) Pulmonary embolism (C) Hospital-acquired pneumonia (D) Fluid overload (pulmonary edema) (E) Atelectasis

13. An 88-year-old woman is brought in by ambulance after falling in her house. She complains of right hip pain. On physical exam, her temperature is 99.1°F, blood pressure is 110/75 mmHg, pulse is 110/min, and respirations are 20/min. The leg appears externally rotated and shortened. X-ray shows femoral neck fracture. Which of the following is a complication of femoral neck fracture?

*(A) Avascular necrosis (B) Lumbar radiculopathy (C) Long-term loss of hip abduction (D) Long-term loss of hip adduction (E) Osteosarcoma

21. A 25-year-old man arrives to the ED after a MVC. After primary survey and secondary survey are completed, he was found to have a right-sided pneumothorax, a fracture in the right clavicle, and a fracture of the left mid- shaft humerus. A bruit is heard over the right upper chest. What is the most appropriate next step after placing a chest tube and subsequent chest X-ray to confirm placement of the tube?

*(A) CT angiogram of the chest and arm (B) Non-contrast CT of the shoulder and clavicle (C) MRI study (D) Nerve conduction test to assess for radial nerve injury (E) Echocardiogram

10. Which of the following is most likely to require urgent colectomy?

*(A) Cecal volvulus (B) Sigmoid volvulus (C) Acute diverticulitis (D) Cecal adenocarcinoma (E) Pseudomembranous colitis

6. An 88-year-old woman with multi-infarct dementia undergoes a coronary artery bypass with the left internal mammary artery used as a conduit for three-vessel disease. Her operation is a success, and she is transferred to the ICU to recover. She is receiving opioids for pain relief. Over the next few days, she develops a markedly distended abdomen with no bowel sounds, pain, or rigidity. She has also not had a bowel movement for the past 48 h. Rectal examination does not demonstrate any retained stool. KUB demonstrates markedly distended colon with gas in the rectum, without air fluid levels. What part of the large bowel is the most likely to perforate?

*(A) Cecum (B) Transverse colon (C) Sigmoid colon (D) Rectum (E) There is very little risk of perforation

3. A 55-year-old female restrained driver arrives to the ED following a MVC. She sustained hyperextension of the cervical spine and hit her chest on the steering wheel. Neurologic examination shows spastic paraplegia and loss of pain sensation in both upper extremities. She is able to move both her legs and can differentiate vibratory sensation and light touch in both feet. What is the most likely etiology for her acute condition?

*(A) Central cord syndrome (B) Anterior spinal artery syndrome (C) Brown-Sequard syndrome (D) Cauda equina syndrome (E) Subacute combined degeneration of the spinal cord

9. A 65-year-old male presents to the ED with nausea, vomiting, and severe abdominal pain. Past history is significant for prior sigmoid colectomy for diverticulitis 10 years ago. On physical exam, his temperature is 100.9 °F, blood pressure is 110/80 mmHg, and heart rate is 110/min. His abdomen has a well-healed midline scar and is distended. Bowel sounds are hyperactive with occasional rushes and tinkles. He has marked right upper quadrant tenderness to palpation with guarding. The rest of the abdominal exam is unremarkable. Abdominal series demonstrates one loop of markedly distended small bowel in the right upper quadrant with an air fluid level. No gas is seen in the colon or rectum. Laboratory values demonstrate a WBC count of 18 × 10 3 /μL (normal 4.1-10.9 × 10 3 /μL) with 15 % bands and a serum lactate of 5 mmol/L (normal 0.5-1.6 mmol/L), BUN 30 mg/dL (7-21 mg/dL), and creatinine 1.2 mg/dL (0.5-1.4 mg/dL). Amylase, lipase, and liver chemistries are normal. NG tube and IV fluids are given. What is the next step in the management?

*(A) Exploratory laparotomy (B) Admit for close observation (C) Upper GI with small bowel follow through with barium (D) Upper GI with small bowel follow through with Gastrografi n (E) Right upper quadrant ultrasound

7. In a patient with an isolated head injury and concerns for increased intracranial pressure, which of the following would have the most potential to benefit the patient? (A) Hypertonic (3 %) saline solution (B) Ventilation with permissive hypercapnia (C) Trendelenburg position of bed (D) Intermittent D50 (dextrose) boluses (E) Nitroprusside drip

*(A) Hypertonic (3 %) saline solution (B) Ventilation with permissive hypercapnia (C) Trendelenburg position of bed (D) Intermittent D50 (dextrose) boluses (E) Nitroprusside drip

12. A 6-week-old boy presents with a 6-day history of non- bilious, forceful vomiting. Between episodes of vomiting, the baby feeds vigorously. On physical exam, the baby has dry mucous membranes with a sunken anterior fontanelle. Capillary refill is 2 s. He is otherwise asymp- tomatic without findings on physical exam. Ultrasound is obtained and is shown below. What electrolyte abnor- mality would you expect?

*(A) Hypochloric metabolic alkalosis (B) Hyperkalemic metabolic acidosis (C) Hyponatremic metabolism acidosis (D) Hyperkalemic metabolic alkalosis (E) Hyponatremic metabolic alkalosis

18. A 50-year-old diabetic male presents with severe RUQ pain and fevers. On physical examination, his temperature is 103.5 °F, BP is 100/60 mmHg, and heart rate is 120/min. He has severe tenderness to palpation in the RUQ. WBC is 20 × 103/μL (normal 4.1-10.9 × 103/μL) with 10 % bands, total bilirubin is 1.0 mg/dL (0.1- 1.2 mg/dL), amylase is 90 μ/L (30-110 μ/L), alkaline phosphatase is 90 μ/L (33-131 μ/L), AST is 110 μ/L (normal 5-35 μ/L), and ALT is 140 μ/L (7-56 μ/L). RUQ ultrasound shows gallstones, a normal common bile duct diameter, and a few gas bubbles within the wall of the gallbladder. IV fluids and antibiotics are administered. The next step in the management consists of:

*(A) Immediate cholecystectomy (B) Admit to ICU for 24-48 h of IV antibiotics and careful monitoring (C) Cholecystostomy (D) CT scan of abdomen (E) ERCP

20. Three hours after total thyroidectomy for thyroid cancer, the patient complains of difficulty breathing. On physical examination, the patient has stridor and appears to be in moderate respiratory distress. Examination of the wound demonstrates tense swelling. The next step in the management is: (A) Immediately reopen wound at the bedside (B) Intubation (C) Emergent return to the operating room for wound exploration (D) Check oxygen saturation (E) Send arterial blood gas

*(A) Immediately reopen wound at the bedside (B) Intubation (C) Emergent return to the operating room for wound exploration (D) Check oxygen saturation (E) Send arterial blood gas

10. A 45-year-old man has had hazy vision for the past month, particularly when he is driving at night. He also endorses small rubberlike nodules on the skin of his trunk, back, arms, and legs that are not painful and do not itch. After seeing his ophthalmologist, he is diag- nosed with bilateral cataracts and is scheduled to receive elective cataract surgery. During induction of anesthesia, following intubation, the patient's pressure increases from 110/70 to 200/90 mmHg. PaCO2 is normal as is his pH. His temperature is 101.5 °F. An esmolol drip is immediately instituted, after which BP increases to 220/90 mmHg and an ECG shows T wave inversion. What is the most likely underlying etiology? (A) Intra-abdominal tumor (B) Malignant hyperthermia (C) Thyrotoxicosis (D) Inadequate anesthetic agent (E) Undiagnosed pituitary tumor

*(A) Intra-abdominal tumor (B) Malignant hyperthermia (C) Thyrotoxicosis (D) Inadequate anesthetic agent (E) Undiagnosed pituitary tumor Neurocutaneous nodules Bilateral cataracts Pheochromotyctoma --> NF-1

1. A 32-year-old man presents with progressive frontal headaches. His symptoms started 2 months ago and often wake him up from his sleep. His vital signs are stable, and neurologic examination reveals no focal deficits. MRI brain imaging reveals a mass lesion, and subsequent biopsy is consistent with a type IV astrocytoma. Which of the following is true regarding this patient's illness?

*(A) It is considered the most common primary malignant brain tumor (B) Prognosis is good since the tumor is slow growing (C) The biopsy should demonstrate psammoma bodies (D) This tumor does not cross the corpus callosum (E) Interferon beta and glatiramer acetate are used in the management of this tumor

7. One week after open repair of a large right scrotal hernia, a 45-year-old male returns complaining of severe pain in his right testicle. On physical exam, the testicle appears to be slightly swollen and very tender to palpation. Doppler study demonstrates no flow within the right testicle with normal flow in the left. Which of the following is true about this condition?

*(A) It is most commonly due to thrombosis of the pampiniform plexus (B) Urgent exploration of the right testicle is recommended (C) It is most likely due to transection of the testicular artery (D) It most likely represents testicular torsion (E) The testicle will likely remain permanently enlarged

10. A 45-year-old female undergoes screening mammography which demonstrates an area of suspicious microscopic calcification in her right upper outer breast. Stereotactic-guided biopsy confirms ductal carcinoma in situ (DCIS). Which of the following is true about this condition?

*(A) It should be excised to a negative margin (B) It is considered a marker for malignancy in either breast (C) The cribriform type has a worse prognosis than the comedo type (D) It does not occur in men (E) Radiation therapy is an acceptable alternative to surgical excision

9. A 52-year-old insulin-dependent diabetic man is evaluated for vague epigastric pain, is diagnosed with GERD, and is treated with proton pump inhibitors with resolution of symptoms. In the course of the workup, however, an abdominal ultrasound was performed. No gallstones were seen, but an incidental 12 mm polyp was found within the gallbladder. What is the next best step in management?

*(A) Laparoscopic cholecystectomy (B) Open cholecystectomy (C) Percutaneous gallbladder drainage (D) Endoscopic ultrasound (E) Repeat ultrasound in 6 months

5. Which of the following is the most common brain cancer?

*(A) Lung metastasis (B) Glioblastoma multiforme (C) Colon metastasis (D) Breast metastasis (E) Astrocytoma

15. A 4-year-old boy presents to the emergency department with right lower quadrant pain, nausea, and anorexia. On physical examination, his bowel sounds are absent, and he has marked tenderness in the right lower quadrant with guarding. The remainder of the abdominal exam is negative. Laboratory values reveal a leukocytosis with a left shift. What is the most likely initiating event for his acute condition?

*(A) Lymphoid hyperplasia (B) Fecalith (<- most common in adults) (C) Parasitic infection (D) Enlarged mesenteric lymph nodes (E) Foreign body ingestion

19. Football players are subjected to multiple sports injuries including valgus stressing of the knee, particularly when they get tackled in the lower extremities subjecting the knee joint to increased abduction. Determining injury to the soft tissue structures of the knee may prove to be difficult for non-orthopedic or sports medicine physicians. After a physical exam, what is the most appropriate study to order to help work-up a patient with pain following valgus stressing of the knee?

*(A) MRI (B) CT (C) Radiograph (D) Arthroscopy (E) Nerve conductions studies

7. A 65-year-old female has breast cancer and a remote history of congestive heart failure. Her physician is planning to administer a chemotherapeutic agent that has potential for cardiac toxicity. Which of the following is the most accurate test to measure ejection fraction?

*(A) Multi Gated Acquisition Scan (MUGA) scan (B) Echocardiography (C) Electrocardiogram (D) Coronary angiography (E) Exercise stress test

5. A 55-year-old schizophrenic homeless man arrives to the ED with abdominal pain and vomiting. He reports that the abdominal pain started yesterday and has been worsening. He is afebrile, blood pressure is 122/86 mmHg, and heart rate is 116/min. In the ED he vomits green emesis without blood. His last bowel movement was 48 hrs ago. Physical examination reveals a large scar in his right upper quadrant. On abdominal examination, the abdomen is distended, with hyperactive bowel sounds, and is tympanic to percussion, with mild diffuse tenderness, and no rebound or guarding. WBC is 9 × 10 3 /μL (normal 4.1-10.9 × 10 3 /μL). Abdominal series shows dilated loops of bowel with multiple air fluids levels. After fluid resuscitation, what is the most appropriate next step in management?

*(A) Nasogastric tube suction (B) Laparoscopy (C) Exploratory midline laparotomy (D) Intravenous erythromycin (E) CT scan of the abdomen

1. A 4-week-old boy presents with a 3-day history of forceful vomiting. The mother states that the vomitus only contains partially digested milk. She notes that the infant seems very hungry between feedings and drinks vigorously. Past history is significant for a skin infection for which the infant received oral erythromycin. On examination, the infant appears to be healthy appearing and in no acute distress. The physician feels there may be a small palpable mass in the right upper quadrant, but is not certain. Electrolytes are normal. What is the best way to establish the most likely diagnosis? (A) Plain abdominal x-rays (B) Laparoscopy (C) CT of the abdomen (D) Upper GI study with contrast (E) Ultrasound

*(A) Plain abdominal x-rays (B) Laparoscopy (C) CT of the abdomen (D) Upper GI study with contrast (E) Ultrasound

12. In addition to elevated plasma free metanephrine, a change in what other serum marker can help support the diagnosis of pheochromocytoma? (A) Plasma chromogranin A (B) Plasma superoxide dismutase (C) Malondialdehyde (D) CA 19-9 (E) 5-Hydroxyindoleacetic acid (HIAA)

*(A) Plasma chromogranin A (B) Plasma superoxide dismutase (C) Malondialdehyde (D) CA 19-9 (E) 5-Hydroxyindoleacetic acid (HIAA)

9. A 2-h-old male infant born at 39 weeks gestation is noted to be drooling. Prenatal ultrasound demonstrated polyhydramnios. Attempts at placement of an orogastric tube are unsuccessful as the tube only passes about 10 cm from the lips. What is the most important immediate concern for an infant with this condition? (A) Prevention of aspiration (B) Nutrition (C) Urgent surgical exploration (D) Establishing positive pressure ventilation (E) Confirmatory contrast esophagram

*(A) Prevention of aspiration (B) Nutrition (C) Urgent surgical exploration (D) Establishing positive pressure ventilation (E) Confirmatory contrast esophagram

17. A 40-year-old female presents with moderate epigastric abdominal pain. She has a history of intermittent RUQ pain after eating fatty foods. On physical examination she is afebrile with a heart rate of 100/min and blood pressure of 110/70 mmHg. She has moderate epigastric tenderness to palpation. Laboratory values are significant for a WBC of 11×103/μL (normal 4.1-10.9×103/μL) with 3 % bands, total bilirubin of 1.2 mg/dL (0.1- 1.2 mg/dL), AST of 250 μ/L (5-35 μ/L), ALT of 300 μ/L (7-56 μ/L), alkaline phosphatase of 150 μ/L (33- 131 μ/L), amylase of 1,300 μ/L (30-110 μ/L), and lipase of 1,100 μ/L (7-60 μ/L). RUQ ultrasound shows numer- ous small gallstones, normal gallbladder wall, and a normal common bile duct diameter of 0.4 mm. On the second hospital day, her pain has resolved; she is afebrile and has a normal heart rate, and her WBC count has normalized. The amylase has decreased to 350 μ/L. Optimal management consists of:

*(A) Proceed with laparoscopic cholecystectomy (LC) with intraoperative cholangiogram (IOC) (B) Wait 4-5 more days until amylase has completely normalized, and then proceed to LC with IOC (C) ERCP followed by LC during same hospitalization (D) Schedule for elective outpatient LC with IOC (E) ERCP only

2. A 61-year-old female presents with swelling and red- ness of her entire left breast that has persisted for 4 weeks. On physical exam her temperature is 98.7 °F, pulse is 82/min, blood pressure is 136/78 mmHg, and respirations are 16/min. Her left breast appears larger than her right one. The entire breast is warm, and the skin is edematous. No breast masses are palpable. There is no nipple discharge or rashes. There are several palpable enlarged lymph nodes in her left axilla. Ultrasound and mammography show thickening of the skin but otherwise no masses. Which of the following is the best option for further management?

*(A) Punch biopsy of skin (to distinguish from mastitis) (B) Oral antibiotics (C) Intravenous antibiotics (D) Nonsteroidal anti-inflammatory drugs (E) Incision and drainage

8. A 65-year-old woman returns to clinic for a 3-month follow-up. Three months ago she developed a pruritic, erythematous, ulcerated rash surrounding the areola of her right breast. She tried hydrocortisone 1 % on the lesion at the recommendation of her primary care physi- cian, but the lesion persisted. She has no history of skin diseases in the family. She takes warfarin for atrial fibril- lation. She started a new medication, hydrochlorothia- zide, for hypertension about 3 months ago. Otherwise, she is healthy. What is the best next step in the manage- ment of this patient?

*(A) Punch biopsy of the skin lesion (B) Change hydrocortisone 1 % to triamcinolone to treat eczema (C) Treatment with antibiotics (D) Oral steroid course to treat psoriasis (E) Increase the dose of hydrocortisone Dx: Paget's (type of DCIS)

7. A newborn full-term infant is noted to have several episodes of bilious emesis. He is otherwise stable. Which of the following is the LEAST likely diagnosis? (A) Pyloric stenosis (B) Duodenal atresia (C) Midgut volvulus (D) Hirschsprung's disease (E) Jejunal atresia

*(A) Pyloric stenosis (B) Duodenal atresia (C) Midgut volvulus (D) Hirschsprung's disease (E) Jejunal atresia

22. A 28-year-old pregnant woman suffering from pre- eclampsia is being monitored in the hospital the night before her elective induction when she experiences a generalized seizure. After the successful delivery of a healthy baby boy the next morning, her doctor notices that she is unable to move her right arm. An upper extremity anteroposterior radiograph is taken but does not reveal any abnormalities. What is the most appropriate next step in management?

*(A) Repeat radiographic studies of the upper extremity (B) MRI of the upper extremity (C) CT scan of the upper extremity (D) Operating room (E) Reassurance and observation

10. One day following extensive debridement of the right leg for a necrotizing soft tissue infection (NSTI), a 40-year-old male remains in the ICU, intubated, and requiring 70 % FIO2. White blood cell count has risen from a preoperative level of 16×103/μL (normal 4.1- 10.9×103/μL) to 34×103/μL. Serum lactate has also risen. Which of the following is the best next step in treatment? (A) Second-look operation (B) Amputation of the right leg (C) Broaden antibiotic coverage (D) CT scan of the leg (E) Start pressors

*(A) Second-look operation (B) Amputation of the right leg (C) Broaden antibiotic coverage (D) CT scan of the leg (E) Start pressors

6. A 30-year-old man is admitted to the hospital for severe acute pancreatitis due to alcohol abuse. His hospital course is complicated by transient renal insufficiency. On hospital day 20, the patient complains of increasing epigastric abdominal pain, nausea, and vomiting. On physical examination, he has a fever of 102 °F and a heart rate of 110/min. Abdominal examination reveals marked epigastric tenderness. His lungs are clear bilaterally. WBC count is 14.5 × 103/μL (normal 4.1-10.9 × 103/μL) with 10 % bands. Blood cultures are sent, and fluid bolus is given. What is the next step in management?

*(A) Start intravenous antibiotics (B) CT scan of the abdomen with contrast (C) ERCP with sphincterotomy (D) Exploratory laparotomy for pancreatic debridement (E) Laparoscopy

7. A 45-year-old alcoholic male presents with fevers and right-hand pain. He cannot recall what happened but thinks he may have punched someone in the face in a bar fight two nights earlier. He has tried over-the-counter anti- inflammatory agents, but they have not helped decrease the pain. His temperature is 100.8°F, blood pressure is 132/88 mmHg, and pulse is 78/min. On physical exam, he has a skin break over his second phalanx-metacarpal region. He is holding his second finger in slight flexion. He has a sausage-shaped swelling of the finger, as well as flexor tendon sheath tenderness that extends the entire length of the tendon. His pain increases with passive motion of the finger. What is the most likely diagnosis?

*(A) Suppurative tenosynovitis due to Eikenella corrodens (B) Gout (C) Dupuytren's contracture (D) Suppurative tenosynovitis due to Pasteurella multocida (E) Felon

9. A college athlete presents with shoulder pain. He reports that he was injured during basketball practice 2 days ago when his teammate ran into his shoulder. On physical examination, he has an edematous left shoulder. The clinician passively abducts the patient's shoulders to 90° and flexes to 30° while asking the patient to point his thumbs down. What muscle is the clinician trying to evaluate?

*(A) Supraspinatus (B) Infraspinatus (C) Teres minor (D) Teres major (E) Subscapularis

2. A 65-year-old male is about to undergo an elective inguinal hernia repair. Which of the following findings on history or physical would portend the highest operative risk?

*(A) Systolic, crescendo-decrescendo murmur at the sternal border of the right second intercostal space radiating into neck (B) A history of myocardial infarction 10 years ago (C) Insulin-dependent diabetes mellitus with an elevated HgbA1C (D) Renal insufficiency not yet on dialysis (E) Smoking

8. Following open inguinal hernia repair, a 50-year-old male complains of numbness and burning pain on the scrotum. This most likely represents injury to:

*(A) The genital branch of the genitofemoral nerve (B) The femoral branch of the genitofemoral nerve (C) The ilioinguinal nerve (D) The lateral femoral cutaneous nerve (E) The iliohypogastric nerve The genital branch of the genitofemoral nerve provides sensation to the scrotum and the cremaster reflex. The femoral branch of the genitofemoral nerve provides sensation to the proximal medial thigh. The ilioinguinal nerve provides sensation to the lower abdomen and medial thigh. The lateral femoral cutaneous nerve provides sensation to the lateral thigh as low as the knee. The iliohypogastric nerve supplies the gluteal region.

5. A 20-year-old college football player sustained a knee injury following a tackle. In the ED, his knee is very swollen, and a proper examination is not possible due to pain. Plain X-rays of the knee are negative. He is discharged home with a knee brace and crutches. During follow-up 2 weeks later, the pain and swelling are significantly improved. Which of the following findings would be most consistent with a tear to the posterior cruciate ligament?

*(A) The lower leg sags on passive flexion of the knee at 90° while the patient is supine (B) The lower leg moves forward relative to the knee with forward traction (C) The foot moves laterally when the knee is pushed medially (D) The foot moves medially when the knee is pushed laterally (E) A clicking sensation is appreciated when the knee is flexed and the leg is rotated externally

3. A 36-year-old woman is evaluated for a lump in her right breast that she noticed 5 months ago. She denies any nipple discharge, nipple retraction, or skin changes. She has no family history of breast cancer. On physical exam, the breasts appear normal. Palpation reveals a 1 cm dominant lump in the left upper quadrant that does not appear to be fixed to the surrounding structures. The patient has no other dominant masses in either breast. There is no axillary lymphadenopathy. Mammogram is negative. What is the next step in the management?

*(A) Ultrasound-guided core needle biopsy (B) Fine-needle aspiration (C) MRI (D) Follow-up clinical breast exam in 3 months (E) Genetic testing

10. A 65-year-old man is rushed to the ED by ambulance after he suddenly lost strength and sensation in his left leg and arm. He was hospitalized 2 months ago with a NSTEMI. He is compliant with all of his medications and had been recovering well until the present episode. ECG shows normal sinus rhythm without evidence of ischemia. Chest X-ray is unremarkable. Carotid ultrasounds show <30 % stenosis bilaterally. What is the most likely etiology of the patient's present symptoms?

*(A) Ventricular thromboembolism (B) Septic embolism to the brain (C) Type A dissection involving the right carotid artery (D) Thromboembolism from the left atrial appendage (E) Paradoxical venous thromboembolism

2. An 11-year-old boy presents to his pediatrician because his mother is concerned that he has been limping for the past 3 days. He states that he is limping because of a dull, aching pain in the right groin and thigh that extends down to his right knee. He has no history of preceding trauma. The pain is increased by physical activity. On physical exam, there is a shift of the torso over the affected hip with standing. Which of the following explains this physical exam finding?

*(A) Weakness in hip abduction (B) Weakness in hip flexion (C) Weakness in hip extension (D) Weakness in knee extension (E) Weakness in knee flexion

13. A 2-h-old male born at 39 weeks gestation is noted to have difficulty breathing. Which of the following would most strongly suggest a diagnosis of esophageal atresia with a tracheoesophageal fistula? (A) Worsening respiratory status with feeding (B) Olive-shaped mass palpable in the epigastric region (C) Improvement of respiratory status over the 24 h after birth (D) Chest x-ray showing dilated loops of small bowel in the left hemithorax (E) Scaphoid abdomen

*(A) Worsening respiratory status with feeding (B) Olive-shaped mass palpable in the epigastric region (C) Improvement of respiratory status over the 24 h after birth (D) Chest x-ray showing dilated loops of small bowel in the left hemithorax (E) Scaphoid abdomen

1. A 15-year-old girl develops short gut syndrome following resection of bowel secondary to leiomyosarcoma in the small intestinal wall. She is subsequently placed on long-term total parenteral nutrition (TPN) and is recovering well. A month later, she develops red and inflamed patches of dry and scaly skin around her mouth and eyes. Her hair also begins to thin, and she notices a bad taste when she gets her daily cherry-flavored chloraseptic spray to prevent dry throat. What is the most likely underlying etiology of her skin lesions and thinning hair? (A) Zinc deficiency (B) Copper deficiency (C) Pemphigus vulgaris (D) Chromium deficiency (E) Psoriasis

*(A) Zinc deficiency: malabsorption syndrome (alopecia, red and inflamed patches of dry and scaly skin around eyes/mouth, impaired wound healing) (B) Copper deficiency (C) Pemphigus vulgaris (D) Chromium deficiency (E) Psoriasis

7. Which of the following patients should undergo carotid endarterectomy (CEA)? (A) 70-year-old male with a history of transient pain- less right eye vision loss and 75 % stenosis of the right internal carotid artery (ICA) (B) 70-year-old male with a history of transient pain- less right eye loss and 100 % occlusion of the right ICA (C) 65-year-old female who sustained a stroke 1 week ago, is aphasic, has no motor function in her right arm or leg, and has 80 % left ICA stenosis (D) 68-year-old male who, 4 days ago, became acutely dizzy and passed out with 60 % right ICA (E) 72-year-old male with an irregularly irregular rhythm who presents with left-sided weakness for the last several hours and has a 40 % right ICA stenosis

A 70 yo male with hx of transient painless right eye vision loss and 75% stenosis of right internal carotid artery

21. A 38-year-old obese construction worker arrives to the trauma bay after accidentally getting struck by a bull- dozer at his job site. In the ED, his mental status is altered, with a GCS of 10. His blood pressure is 80/66 mmHg with a pulse of 112/min. He is given 2 liters of intravenous fluids, but his blood pressure and pulse remain the same. A FAST exam is inconclusive. A portable chest x-ray is negative, and a pelvic x-ray demonstrates bilateral pubic rami fractures. What is the best next step in management? (A) Diagnostic peritoneal lavage (DPL) (B) Pelvic angiography with possible embolization (C) Exploratory laparotomy (D) Head CT scan (E) Abdominal CT scan

A DPL Between DPL and pelvic angiography, pelvic angiography follows DPL Head CT/abdominal CT not good because hemodynamically unstable

3. A 70-year-old male with a history of smoking presents with vague upper abdominal and epigastric pain and coffee ground emesis. On EGD, a small ulcer is found in the stomach, with evidence of a large submucosal mass underneath the ulcer. Biopsy of the area is negative. CT scan confirms a 4-cm homogeneous, well-circumscribed, submucosal mass in the greater curvature of the stomach. What does this mass most likely represent? (A) Gastrointestinal stromal tumor (B) Gastric adenocarcinoma (C) Gastric lipoma (D) Metastatic carcinoma (E) Solitary fibrous tumor

A GIST (mesenchymal tumors that most often occur in the stomach and small intestine), as they grow, they can cause pressure necrosis, leading to erosion of the gastric mucosa and GI bleeding. CT scan: homogeneous, well circumscribed mass. c-KIT tx: surgical resection, followed by imatinb mesylate, a selective tyrosine kinase inhibitor gastric adenocarcinoma starts in the mucosa. unlikely to be biopsy negative.

20. A 62-year-old male presents to the physician with pain in his buttocks that comes on during his evening walks with his wife, forcing him to stop and rest. He also confides in you that he has been having difficulty maintaining an erection leading to marital problems. Physical exam is significant for absent femoral and distal pulses. What is the most likely diagnosis? (A) Leriche syndrome (B) Acute occlusion of the infrarenal aorta (C) Extensive atherosclerosis of the superficial femoral artery (D) Spinal stenosis (E) Bilateral hip osteoarthritis

A Leriche syndrome (triad: impotence, buttock/thigh claudication, absent femoral pulses)

10. A 10-year-old boy presents to his doctor for follow-up 2 weeks after having an upper respiratory tract infection (URI). He complains of pain in his scrotum. It has been bothering him for the past 3 days but is more painful today. A scrotal ultrasound demonstrates an enlarged and rounded epididymis. His scrotal skin appears thickened. What other finding(s) would you suspect in this patient's history and physical? (A) Skin lesions and abdominal pain (B) Recent weight loss, night sweats, and fevers (C) Recent epididymitis (D) Sores in the mouth and swelling of the eyes (E) Painful urination

A Skin lesions and abdominal pain HSP- infrequently scrotal pain

6. A 40-year-old man falls down approximately three stories in an attempt to commit suicide. EMS arrives on scene within 5 min, and he is rushed to the ED but loses vitals in the field and is dead on arrival (DOA). What is the most likely cause of death? (A) Thoracic aortic transection (B) Tension pneumothorax (C) Abdominal aortic transection (D) Ruptured spleen (E) Pulmonary artery transection secondary to a jagged rib edge

A TAA mechanism: rapid deceleration injury

6. A 50-year-old male presents to the ED stating that he passed a large amount of maroon stool earlier in the day. He is currently not passing any stool. He has not vomited. He denies abdominal pain or weight loss. He also has a history of nosebleeds. He takes no medications and does not drink alcohol. Physical examination is only significant for small red nodules on his lips. Abdominal examination is unremarkable. Upper endoscopy is negative. Which of the following is most likely to localize the site of the GI bleeding? (A) Capsule endoscopy (B) CT angiogram (C) Lower endoscopy (D) Formal angiography (E) Technetium-labeled red blood cell scan

A capsule endoscopy Dx: hereditary hemorrhagic telangiectasia (HHT) or Osler-Weber Rendu, autosomal dominant disorder cxed by epistaxis history, red nodules of face, lips, AV malformations, , and f'amily history. GI bleeding often in stomach and small bowel. Upper endoscopy is negative in the vignette, then you do capsule endoscopy to look further !

14. A 44-year-old female is brought to the ED after a vehicle sideswiped her while she was on her motorcycle. She is complaining of right leg and knee pain. Initial survey reveals stable airway, breathing, and circulation. On exam, her heart has regular rate and rhythm; lungs are clear. No abdominal tenderness of rebound. There is 0/5 motor strength in dorsiflexion of the foot and decreased sensation to pinprick at the dorsum of the foot. The extremity is warm and well perfused, and dorsalis pedis, posterior tibialis, and popliteal pulses are 2+ bilaterally. What is the most likely etiology of her neurologic deficits? (A) Common peroneal nerve injury (B) Compartment syndrome (C) Arterial thrombosis (D) Deep peroneal nerve injury (E) Pain from the fracture

A common peroneal nerve presentation of peroneal nerve injury at fibular neck: acute foot drop (difficulty dorsiflexing of foot against gravity) + numbness of foot injury to deep peroneal nerve: numbness in first web space.

11. A 62-year-old man with atrial fibrillation presents to the ED with a painful right lower leg. He has refused warfarin in the past. His physical exam is significant for an irregularly irregular heart rate and a painful right leg that is cool to touch with absent distal pulses. Pulses in the left foot are normal. He has significant motor weakness and sensory deficit in the right foot. Duplex scan reveals an occlusion of the right popliteal artery. He receives heparin and undergoes open surgical embolectomy. Following the procedure, his motor and sensory deficit dramatically improves. The next day, he experiences intense pain in the right calf. His right calf is swollen and tense, and the pain is worsened with passive dorsi and plantar flexion of his right foot. He has palpable distal pulses. What is the most likely underlying etiology for his acute condition? (A) Interstitial edema (B) Recurrent embolization (C) Deep vein thrombosis (DVT) (D) Atherosclerotic plaque (E) Lymphedema

A interstitial edema compartment syndrome

16. A 32-year-old female is stabbed in the right lateral neck 1 cm above the clavicle. There is an expanding hematoma in her neck, and she is having great difficulty speaking. Breath sounds are absent on the right. Subcutaneous air is noted in her neck. What is the next step in management? (A) Intubation (B) Chest tube placement (C) Duplex ultrasound of the carotid (D) Operative repair (E) Esophagoscopy

A intubation esophagoscopy should not be performed in the setting of vascular injury

5. A 78-year-old man arrives to the ED with colicky flank pain for the past 4 days that is now accompanied by nausea, vomiting, fever, and hematuria. Past medical history is significant for congestive heart failure and prior myocardial infarction. On physical examination, the patient has a blood pressure of 100/60 mmHg, temperature of 101 °F, and a heart rate of 110/min. Urinalysis reveals 150 RBC/hpf and 20 WBC/hpf. Laboratory tests demonstrate a WBC of 15 × 103/μL (normal 4.1-10.9 × 103/μL) with 10 % bands. Imaging demonstrates a 10-millimeter stone lodged in the ureterovesicle junction with dilation of the right renal calyx. Broad-spectrum antibiotics are administered intravenously. What is the best next step in management? (A) Percutaneous nephrostomy tube (B) Open nephrostomy (C) Shock wave lithotripsy (D) Placement of a ureteral stent (E) Admit to ICU for close monitoring

A percutaneous nephrostomy tube (more expeditious than placement of ureteral stent, as ureteral stent is more time consuming, esp for septic patient!)

8. A 70-year-old male presents with right calf pain after walking three blocks, forcing him to stop and rest. He denies pain at rest in his foot. He smokes one pack per day. His pulse is 74/min, respiratory rate is 16/min, and blood pressure is 136/86 mmHg. Physical examination reveals diminished pulses in his right foot. The ankle- brachial index on the right leg is 0.7. The next step in the management is: (A) Smoking cessation (B) CT angiogram (C) MR angiogram (D) Formal angiogram (E) Stenting of iliac artery

A smoking cessation

21. A 65-year-old male presents with new onset claudication. An ABI confirms that he has peripheral arterial disease (PAD). Which of the following would provide the least amount of benefit to the patient? (A) Warfarin (B) Aspirin (C) Statin (D) Cilostazol (E) Clopidogrel

A warfarin

2. Preoperative medical optimization for a patient with a pheochromocytoma routinely includes: (A) Octreotide drip for 24 h before surgery (B) Controlofhypertensionwithbeta-blockadeasfirst- line agent (C) Control of hypertension with alpha-blockade as first-line agent (D) Metyrosine (E) Diuretics for blood pressure management

A) Octreotide drip for 24 h before surgery (B) Controlofhypertensionwithbeta-blockadeasfirst- line agent *(C) Control of hypertension with alpha-blockade as first-line agent (D) Metyrosine (E) Diuretics for blood pressure management

11. A newborn male is found to have the majority of his small bowel eviscerated through an abdominal wall defect. The umbilicus appears to be intact. There is no membrane covering the bowel. The most important immediate risk to an infant with this condition is related to: (A) Sepsis (B) Respiratory compromise (C) Cardiac anomalies (D) Dehydration (E) Urinary obstruction

A) Sepsis (B) Respiratory compromise (C) Cardiac anomalies *(D) Dehydration (E) Urinary obstruction

15. An obese 11-year-old presents to his pediatrician because his mother is concerned that he has been limping for the past day. He states that he is limping because of dull pain in his right knee. He has no history of preceding trauma. The pain is increased by physical activity. On physical exam, there is tenderness to palpation at the anterior hip along with limitation in internal rotation of the hip. Knee exam is normal. Plain films show subluxation of the right femoral head. Which of the following is the most appropriate definitive treatment?

A) Supportive therapy with rest and ibuprofen (B) Aspiration of the synovial fluid along with appropriate antibiotic therapy (C) Weight loss program *(D) Operative stabilization (E) Pavlik harness

1. A 52-year old male is brought to the hospital by his wife with complaints of intense pain that started around his right flank and now radiates to his right groin. He said that his urine appears pink. He appears to be in severe pain and is unable to remain still during examination. His abdominal exam is unremarkable. Urinalysis reveals 100 RBC/hpf. IV fluids and analgesics are administered. Which of the following is the most appropriate imaging? (A) Helical CT scan of the abdomen and pelvis without contrast (B) Helical CT scan of the abdomen and pelvis with contrast (C) Upright abdominal X-ray (D) Intravenous pyelogram (IVP) (E) Renal ultrasound

A. CT abdomen/pelvis without contrast (or KUB, supine xray of abdomen)

10. Which of the following findings on urinalysis would most strongly support the diagnosis of acute tubular necrosis? (A) Muddy brown casts (B) Urine osmolarity > 500 (C) Bland urine sediment (D) Red cell casts (E) White cell casts

A. muddy brown casts

1. A 60-year-old male presents with gnawing epigastric pain that has been present for 8 weeks. The pain seems to get worse with eating. He denies nausea, vomiting, or early satiety. He has lost 10 lb and he attributes this to a poor appetite. He denies black or bloody stools. Physical examination is unremarkable. What is the next study to be ordered? (A) Barium swallow (B) CT scan (C) EGD (D) 6-week trial of proton pump inhibitor (E) Testing for H. pylori

ALARM signs: anorexia, recent onse of progressive sxs, melena/hematemesis/swalloing problems -> must rule out gastric cancer with upper endoscopy! C: EGD

2. A 56-year-old man is seen in the ED for onset of severe pain and numbness in his left leg that began 30 min ago. He has a past medical history significant for hypertension, diabetes, and hyperlipidemia. Physical exam reveals a temperature of 98.3 °F, blood pressure of 134/74 mmHg, pulse of 89/min, and respiratory rate of 16/min. Heart is irregularly irregular with no murmurs appreciated. On extremity exam, there is absent pulses in the left femoral, popliteal, and pedal arteries. On the right, all pulses are 2+. The left foot is cool to touch as compared to the right. Sensation to pinprick is decreased on the left dorsum of the foot compared to the right. What is the most likely etiology of these findings? (A) Arterial thrombosis (B) Peripheral neuropathy (C) Arterial embolism (D) Cerebrovascular accident (E) Venous thrombosis

Arterial embolism (C)

Peripheral artery disease

Atrophy of affected muscles, decreased pulses, decreased hair growth, shiny skin, claudication

27. A 25-year-old male suffers a GSW to his right mid- thigh. On physical examination, there is no hematoma, no palpable thrill, and no bleeding from the wound. He has diminished but present pedal pulses on the right and normal pulses on the left. Neurological exam is normal. Ankle-brachial index on the right is 0.8 and 1.0 on the left. What is the next step in the management? (A) Surgical exploration of the leg (B) CT angiography (C) Formal angiography (D) Observation (E) Systemic heparinization

B CT angio Penetrating injury to extremities should be evaluated for neuromuscular injury Penetrating injury -> check for vascular injury (pulsatile bleeding, expanding hematoma -> if no vascular injury, check ABI -> if less than 0.9, worry about arterial injury and do CT angio -> if normal ABI then observe-> if CT angio is equivocal then do normal angio

15. A 5-year-old girl arrives to the ED with complaints of nausea, vomiting, and abdominal pain for the past day. She has no significant past medical history, but her mother reports that she was involved in a MVC about a month ago. She was restrained in a car seat and had blunt trauma to her abdomen. She had no complaints at the time. Her vital signs were normal, and she was subsequently discharged a few hours later. Her blood pressure is currently 112/82 mmHg, pulse is 90/min, and respiratory rate is 28/min. Her chest x-ray is shown above (Fig. A.1). What is the most likely diagnosis? (A) Gastroenteritis (B) Diaphragmatic hernia (C) Delayed splenic rupture (D) Hemothorax contusion (E) Pneumonia

B Diaphragmatic hernia (can have delayed presentation)

5. A 40-year-old policeman is brought to the ED having suffered burns after helping to rescue a woman from a burning warehouse. His temperature is 99.8 °F, blood pressure is 100/70 mmHg, pulse is 95/min, and respiratory rate is 24/min. On physical examination, he has 40 % total body surface area deep partial and full-thickness burns to his face, arms, and back as well as a circumferential burn of his neck. He has singed nasal hairs, and there is carbonaceous sputum coming out of his mouth. His lungs are clear to auscultation bilaterally. ECG demonstrates premature ventricular contractions. What is the most appropriate next step in management? (A) Broad-spectrum antibiotics (B) Endotracheal intubation (C) IV fluid resuscitation (D) Cardiac enzymes and serial ECG (E) Bronchoscopy

B ET intubation *circumferential burn of neck

9. A 40-year-old man is in a head-on MVC with a drunk driver on the freeway and is brought to the ED. He has a dark bruise from his seat belt across the left side of his neck. On physical examination, he is neurologically intact. However, his left eyelid is drooping, and his left pupil is constricted as compared to his right. CT scan with contrast demonstrates dissection of the left internal carotid that extends into the base of the skull. CT of the head and abdomen are negative. Which of the following would be the most appropriate management? (A) Left neck exploration (B) Intravenous heparin administration (C) Carotid stenting (D) Thrombolytic therapy (E) Observation

B IV heparin administration Dissection can occlude artery

11. What is the most common cause of an esophageal perforation? (A) Boerhaave syndrome (spontaneous perforation) (B) Iatrogenic (C) Foreign body (D) Esophageal cancer (E) Trauma

B Iatrogenic

8. A 64-year-old female was recently discharged following a morbid obesity surgery (gastric sleeve resection). Her postoperative course was complicated by a DVT in her left leg, for which she received intravenous heparin and discharged with oral warfarin. She now presents with an area of discolored, purplish skin on her right thigh that began the prior day and is extremely painful. On physical exam, she is afebrile with a normal blood pressure and heart rate. There is a 6 × 6 cm area of pur- plish black skin over her right anterior thigh, and another smaller area on her right calf. On laboratory exam, her WBC count and serum glucose are normal. What is the most likely cause? (A) Vitamin K deficiency (B) Protein C deficiency (C) Heparin (D) Thrombocytopenia (E) Unrecognized hemophilia

B Protein C deficiency warfarin induced skin necrosis (worse in people with protein c deficiency)

12. A 26-year-old man that recently emigrated from Sudan to the United States arrives to your office complaining of red urine. He does not experience any pain but says that for the last week his urine has appeared red. He also reports night sweats, fevers, and losing 10 pounds over the last 2 months without a change in his appetite. Cystoscopy is performed, and biopsies are taken of a tumor that appears nodular with a plaque-like and irregular surface. What is most likely to be seen on microscopic analysis? (A) Urothelial carcinoma (transitional cell carcinoma) (B) Squamous cell carcinoma (C) Adenocarcinoma (D) Mucinous (colloid) carcinoma (E) Small cell carcinoma

B Squamous Cell Carcinoma (Sudan, Egypt, Tanzania) they have schistosomiasis- chronic granulomatous cystitis secondary to the paraside -> squamous metaplasia -> SCC Most common type of bladder cancer in the US is urothelial (A)

20. A 30-year-old unrestrained driver is brought in by paramedics after a high-speed MVC. In the ED, his heart rate is 110/min, blood pressure is 104/75 mmHg and decreases to 92/68 mmHg during inspiration. His tachycardia and hypotension persist despite aggressive fluid resuscitation. He appears pale, and his neck veins are distended. He has multiple bruises on his chest and abdomen. His chest x-ray is unremarkable. What is the most likely diagnosis? (A) Aortic transection (B) Cardiac tamponade (C) Severe lung contusion (D) Tension pneumothorax (E) Diaphragm injury

B cardiac tamponade

13. A 10-year-old boy presents to the ED with severe abdominal pain after falling over his bicycle handles while attempting a trick and sustaining blunt injury to the abdomen. A CT scan shows oral contrast extravasation into the retroperitoneum that is coming from the posterior aspect of the duodenum. Which of the following is the best management recommendation? (A) Laparoscopy (B) Exploratory laparotomy (C) Observation (D) Upper endoscopy to confirm injury (E) CT-guided drainage

B exploratory laparotomy assess for other injuries bc isolated duodenal injury is uncommon

24. A 63-year-old male comes to the physician's office for a nonhealing ulcer on his right great toe and intermittent calf pain. His calf pain is brought on by walking for 10 minutes and relieved by rest. He wakes up at night with pain in his great toe and has to get up to relieve it. He has a past medical history significant for hypertension, hyperlipidemia, and diabetes. He has a 30-pack- year smoking history. On physical exam, his temperature is 98.4 °F, pulse is 80/min, blood pressure is 139/82, and respirations are 16/min. His LDL is 70 mg/dL. Exam is significant for absent dorsalis pedis and popliteal pulses on the right. The ulcer appears dull gray without purulence. He has diminished sensation to light palpation in both feet. Ankle-brachial index is 0.3 on the right and 0.7 on the left. What is the most appropriate next step in management? (A) Diabetic shoe to offload pressure (B) Formal angiography of the lower extremities (C) Tight glucose control (D) Start oral antibiotics (E) Start heparin

B formal angiography of lower extremities dx: ischemic rest pain and nonhealing ulcer --> limb threatening! therefore imaging for intervention

3. A 41-year-old patient presents to the emergency department following a stab wound to the chest, just above the left nipple line. On initial exam, his blood pressure is 94/70 mmHg, and respiratory rate is 16/min. He has distended neck veins, and his heart sounds are muffled. A FAST exam demonstrates fluid in the pericardial sac. What is considered the first sign of this condition? (A) Electrical alternans (B) Impaired diastolic filling (C) "Water-bottle" shape on chest radiograph (D) Hypotension (E) Distended neck veins

B impaired diastolic filling

15. A 68-year-old female presents to the ED 1 week after total hip replacement. She became suddenly short of breath 2 h ago. Her vitals include a blood pressure of 100/60 mmHg, heart rate of 120/min, and respiratory rate of 30/min. On physical examination, lung sounds are clear. Chest x-ray is normal. Arterial blood gas shows a pH of 7.53, PaCO2 28, PaO2 70, and HCO3 25. Oxygen is given by nasal cannula. Which of the following is the next best step in the management? (A) Spiral CT angiogram of the chest (B) Intravenous heparin (C) Intravenous thrombolytic infusion (D) Venous duplex scan of both legs (E) Echocardiogram

B iv heparin

15. A 74-year-old male presents to the ED with loss of vision in his right eye. Which of the following findings is most suggestive of a nonsurgical diagnosis? (A) Bruit on auscultation of the right carotid artery (B) Jaw pain with chewing (C) Eighty percent stenosis of the right carotid artery on CT angiogram (D) Left hand weakness (E) Atrial mass on echocardiogram

B jaw pain with chewing (GCA managed with steroids)

2. Which of the following is the strongest risk factor for gastric cancer? (A) Type A blood (B) H. pylori infection (C) Smoking (D) Familial adenomatous polyposis (E) Family history

B. H pylor infection

13. A 25-year-old woman is referred to an ENT surgeon with complaints of recurrent nosebleeds for the past month. She also reports that she has been easily bruising with minor trauma and her last menstrual period required double the change of tampons she typically uses. She does not report any blood in stool or urine. Her physical exam is significant for mild gingival bleeding and scattered bruises on her arms and legs. Her laboratory exam is significant for an isolated thrombocytopenia of 13,000 (normal 150,000-400,000). Bone marrow biopsy is normal other than an increase in megakaryocytes. What is the best initial management for this condition? (A) Platelet transfusion (B) Corticosteroids (C) Splenectomy (D) Plasmapheresis (E) Intravenous immunoglobulin (IVIG)

B. corticosteroids not platelet transfusion (ITP [<50,000] is a consumptive process)

8. A 15-year-old boy arrives to the ED 4 h after experiencing a sudden onset of right testicular pain while playing soccer. He does not recall any specific trauma to his testicle during the game. He also reports nausea and vomiting. Physical examination reveals a tender and swollen right testicle that is displaced superiorly. The testicle appears to be lying transversely. He has an absent cremasteric reflex on the right. The left testicle is normal in location and is non-tender. What is the next step in management? (A) Ultrasound (B) Color Doppler ultrasound (C) Right orchiopexy (D) Bilateral orchiopexy (E) Scrotal support and nonsteroidal anti-inflammatory drugs

Bilateral orchiopexy (D)

2. A 23-year-old male is rushed to the ED by paramedics after sustaining a gunshot wound to the lateral neck at the level of the thyroid cartilage. The patient is hemodynamically stable and is able to speak. Physical exam shows no signs of hematoma, pulsatile bleeding, thrill, or bruit. Which of the following is the next step in management? (A) Surgical exploration (B) Wound closure (C) CT angiography (D) Intubation (E) Formal angiography

C CT angio

7. A 45-year-old male presents with a new scrotal mass on the right side that he noticed a week earlier. It is non-tender. He reports no history of trauma. When the patient is standing, the scrotal mass is separate from the testicle and epididymis, feels like a "bag of worms," and is soft, compressible, and non-tender. With the patient supine, the mass is unchanged. The left side is normal. What is the most appropriate next step in the management? (A) Scrotal support and NSAIDs (B) Surgical intervention (C) CT of the abdomen (D) Reassurance and observation (E) Ultrasound-guided biopsy of the mass

C CT of abdomen (to check for masses) RIGHT sided variocele likely from RCC impinging on IVC. Remember, left side varioceles much more common! :)

10. A 22-year-old male arrives to the ED by paramedics with a gunshot wound in the RUQ of his abdomen. He is anxious and complains of pain near his wound. His temperature is 99.1 °F, blood pressure is 114/78 mmHg, and pulse is 90/min. His abdomen is soft, and he has no rebound or guarding. A portable chest x-ray is normal, and nasogastric tube (NGT) demonstrates clear fluid with no blood. His rectal examination shows no blood. What is the most appropriate next step in management? (A) Serial physical examination (B) Exploratory laparotomy (C) CT scan of the abdomen (D) Diagnostic peritoneal lavage (E) FAST exam

C CT scan of abdomen for hemodynamically stable patients with penetrating abdominal injury Fast exam more appropriate for blunt trauma

18. In a patient presenting with acute limb ischemia in the right leg, what is the first structure to develop ischemic changes? (A) Fat (B) Nerve (C) Muscle (D) Skin (E) Bone

C Muscle (tolerant of ischemia for ~4 hrs)

3. A 68-year-old male with a 50-pack-year history of smoking presents with 2 h of left arm weakness. Physical exam shows 1+ strength in the left upper extremity and 3+ strength in the proximal left lower extremity. Which of the following is the next step? (A) Doppler ultrasound of the right carotid artery (B) Echocardiogram (C) Non-contrast CT brain (D) ECG (E) CT angiogram of the neck

C Noncontrast CT brain (check for hemorrhage) This is ACUTE care (doppler would be to check for source later)

5. A 65-year-old male comes to the physician for a routine physical exam. He has a past medical history significant for hypertension and hyperlipidemia. He is on hydrochlorothiazide, aspirin, and a statin. He has a 10-pack- year smoking history but quit over 30 years ago. Which of the following screening tests is appropriate for this male? (A) CT scan of the abdomen (B) Duplex scan of the carotid arteries (C) Abdominal ultrasonography (D) Pulmonary spirometry (E) Ankle-brachial index (ABI)

C abdominal US (USPSTF recommends 1 time screening for abdominal aortic aneurysm by US in men ages 65-75 with smoking history)

7. A 50-year-old female undergoes right hepatic lobectomy for metastatic colon cancer. The operation took 6 h and was associated with significant bleeding. As a result, she was markedly hypotensive throughout the operation and received multiple units of blood products. She was admitted to the surgical ICU for further management. She received one dose of prophylactic antibiotics preoperatively, but is otherwise not receiving any medications. On postoperative day 1, her urine output is only 10 cc/h for 5 h. Her serum creatinine has risen from 1.2 to 2.0 mg/dl. Her blood pressure is 140/80 mmHg, and heart rate is 100/min. Lungs are clear to auscultation. Urine Na is 44 mEq/L, and FENa is 3.1 %. Which of the following is the most likely etiology? (A) Hypovolemia (B) Acute interstitial nephritis (C) Acute tubular necrosis (D) Cardiogenic shock (E) Obstructed urinary catheter

C acute tubular necrosis

10. A 65-year-old male presents with melena. He reports no abdominal pain and no vomiting. He denies a prior similar history. In the ED, he is tachycardic to 120/min with a blood pressure of 112/80 mmHg. Hemoglobin and hematocrit are 9.2 g/dL (normal 13-16 g/dL) and 28 % (40-52 %), respectively. Past history is significant for an aortobifemoral bypass 5 years earlier for severe claudication. Nasogastric (NG) tube aspiration returns clear fluid. The patient is admitted to the ICU for resuscitation, the melena ceases, and the hematocrit stabilizes. Upper endoscopy is negative, with no stigmata of bleeding. Following a bowel prep, colonoscopy is also negative. CT scan of the abdomen reveals gas around the proximal aortic graft. Which of the following is true about this condition? (A) Formal angiography is likely to demonstrate the source of bleeding (B) Gas around the aortic graft is not considered pathologic (C) The aortic graft will likely need to be removed. (D) Rebleeding is rare (E) This represents an occult bleed

C aortic graft will need to be removed Gi bleeding in association with aortobifemoral bypass is due to an aortoenteric fistula until proven otherwise!

2. A 30-year-old male is involved in a high-speed MVC. He was wearing his seatbelt. In the ED, he has a strong odor of alcohol. Blood pressure is 120/70 mmHg and heart rate is 80/min. His abdomen appears distended and mildly tender diffusely. The pelvis is stable. On rectal exam, the prostate feels normal. There is no blood at the urethral meatus. On insertion of a Foley catheter, there is gross hematuria. CT scan of the abdomen and pelvis with oral and IV contrast reveals a large amount of free fluid and contrast in the peritoneum. The liver and spleen appear to be normal as does the pelvis. What part of the genitourinary tract is most likely injured? (A) Ureter (B) Base of the bladder (C) Bladder dome (D) Renal hilum (E) Urethra

C bladder dome (intraperitoneal organ) ?reminder: why do retroperitoneal bladder injuries managed nonoperatively iso pelvic fracture?

14. A 40-year-old alcoholic presents to the ED with a markedly swollen right forearm that is diffusely tender. He states that following an alcohol and heroin binge, he fell asleep on his arm for 12 h. He woke up to find his hand completely numb and unable to move it. On physical exam, he has normal brachial and radial pulses. His heart has a regular rate and rhythm. He is unable to extend his wrist when the hand is palm down. ECG reveals peaked T waves, and CPKs are 20,000 IU/L (normal 60-400 IU/L). What is the next step in management? (A) Propranolol (B) Insulin+dextrose (C) Calcium gluconate (D) Kayexalate (sodium polystyrene) (E) Furosemide

C calcium gluconate

4. A pregnant woman in her second trimester arrives to the ED after a minor MVC. She has no injuries or complaints but is worried that her pregnancy is in danger. She has a nonstress test that shows two accelerations of fetal heart rate, each at least 15 beats per minute above baseline and lasting at least 15 s. She has no contractions, vaginal bleeding, or abdominal pain. A FAST exam is negative. What is the next best step in management? (A) Monitor the patient overnight (B) Biophysical profile (C) Discharge and follow-up in 2 weeks (D) CT of the abdomen (E) MRI of the abdomen

C discharge and f/u in 2 wks

30. A 28-year-old man arrives to the ED following a high- speed MVC. He is in severe pain and breathing rapid shallow breaths. His blood pressure is 80/60 mmHg, heart rate is 120/min, and respiratory rate is 30/min. A segment of his right anterolateral chest wall exhibits paradoxical inward motion on inspiration. Despite supplemental oxygen, the respiratory rate remains the same. Breath sounds are equal bilaterally. The trachea is mid- line. What is the next best step in management? (A) Two liter bolus of normal saline (B) Insert a right needle thoracostomy (C) Endotracheal intubation (D) Transfuse two units of O-negative blood (E) Place tube thoracostomy (chest tube) on the right

C et tube

1. A 55-year-old woman arrives to the emergency department with a 3-h history of severe epigastric pain that has been progressing in severity. She describes it as sharp, diffuse, and constant. Her past medical history is significant for chronic atrial fibrillation for which she takes warfarin. Her temperature is 100.6 °F, blood pressure is 102/66 mmHg, and pulse is 98/min and irregular. Physical examination reveals a diffusely tender and rigid abdomen, with guarding and rebound. An upright abdominal x-ray demonstrates free air under the right hemidiaphragm. Her laboratory exam findings include a hemoglobin 10.2 g/dl (normal 12-15 g/dl), platelets 110,000 (150,000-400,000), INR 2.5, and PTT 18 s (18-28 s). Which of the following is the best way to manage the INR in this patient? (A) Hold warfarin and allow INR to autocorrect (B) Oral vitamin K (C) Fresh frozen plasma (D) Intravenous vitamin K (E) Cryoprecipitate

C fresh frozen plasma

2. What is the most common CXR finding in a patient with pulmonary embolism (PE)? (A) Consolidation of one lobe (B) Fluffy bilateral infiltrates (C) Normal (D) Hampton's hump (a wedge-shaped, pleural-based consolidation) (E) Westermark's sign (a focus of oligemia leading to collapse of pulmonary vessel)

C normal

22. A 25-year-old male is at a pool party and is heavily intoxicated. He dives into the shallow end of the pool and is subsequently found to be floating face down in the pool. He is rushed to the ED by paramedics in a cervical collar. In the ED he opens his eyes, nods his head appropriately to questions, and his pupils are equally round and reactive to light. However, he is not moving his arms or legs. There is no evidence of external bleeding. His blood pressure is 85/45 mmHg, and his heart rate is 70/min. Which of the following would most likely be seen in association with the injury described? (A) Low cardiac output (B) Elevated SVR (C) Priapism (D) Parasympathetic blockade (E) Thoracic spine injury

C priapism - presenting sign of acute spinal injury sequelae of neurogenic shock (high cervical spinal cord injury) hypotension, inappropriately normal heart rate (or bradycardia) unopposed parasympathetic blockade

22. A 30-year-old male presents with a 4-month history of a painful right great toe. In addition, he states that when he walks about two blocks, his right foot hurts and he has to stop to rest. He has a 20 pack year smoking history. His pulse is 74/min, respirations are 18/min, and blood pressure is 126/80 mmHg. His total cholesterol is 160 mg/dl. His serum calcium is 9.4 mg/dL, and on physical examination, he has normal femoral and popliteal pulses bilaterally, but no palpable dorsalis pedis or posterior tibial pulses in either foot. The big toe is slightly bluish and tender to palpation. Which of the following is true about this condition? (A) It responds well to stenting (B) A bypass will be curative (C) Smoking cessation is the cornerstone of management (D) It is unlikely to be due to arterial occlusion (E) It most likely represents early atherosclerosis

C smoking cessation is cornerstone Dx: Buerger's disease (thromboangiitis obliterans)- non atherosclerotic vascular occlusive disease seen in young, mostly male smokers. stenting/surgical bypass ineffective, as occlusions involve most distal arteries. only effective tx is smoking cessation! associated with high rates of amputation, esp if patient continues smoking.

13. A 75-year-old male is recovering in the ICU from an open repair of a ruptured abdominal aortic aneurysm. On post-op day 1, he complains of abdominal pain and tenderness over the left lower quadrant, without rebound or guarding. On post-op day 2, he develops bloody diarrhea. Physical exam reveals a blood pressure of 129/79 mmHg, pulse of 100/min, temperature of 99.9 °F, and respirations of 16/min. What is the most likely etiology of his abdominal pain and bloody diarrhea? (A) Colonic ischemia (B) Clostridium difficile colitis (C) Diverticulitis (D) Inflammatory bowel disease (E) Colon cancer

Colonic ischemia (A)

Weber test: fork in the middle of your head

Conductive hearing loss: sound is best heard in the abnormal ear Sensorineural hearing loss: sound is best heard in the normal ear

4. A 59-year-old male comes to see his primary care physician endorsing a 10-month history of increasing bilateral calf pain with exercise. He initially only felt discomfort after walking six blocks but now it occurs after only one block. The pain disappears with rest. His past medical history is significant for hypertension and hypercholesterolemia, for which he takes lisinopril and a statin. He has smoked two packs per day for the past 40 years. His physician orders a complete work-up and decides the patient most likely is suffering from claudication secondary to peripheral arterial disease (PAD). Which of the following findings would be the best way to support the diagnosis of PAD? (A) History of pain in legs with exertion (B) Diminished peripheral pulses on physical examination (C) A combination of history and physical examination (D) An ankle-brachial index < 0.9 (E) Arteriographic evidence of atherosclerotic plaques

D ABI <0.9

11. A 65-year-old homeless man with poorly controlled diabetes presents to urgent care with severe pain and swelling in his left leg. Vital signs include a temperature of 102 °F, blood pressure of 132/78 mmHg, and heart rate of 102/min. On physical exam, he appears to be tachypneic. His left leg appears tense, and the skin is warm and red over his thigh and is tender to palpation. There is an area of the skin on the thigh that has a violaceous color with blistering. Laboratory testing demonstrates a WBC 22 × 103/μL (normal 4.1-10.9 × 103/μL) with neutrophilic predominance and a serum Na of 128 mEq/L(137-145 mEq/L). What is the best next step in treatment? (A) Immediate IV heparin (B) Venous duplex scan of left leg (C) CT pulmonary angiogram (D) Blood cultures, broad-spectrum antibiotics, and urgent surgical debridement (E) Blood cultures and broad-spectrum antibiotics

D Blood cultures, broad spectrum antibiotics, and urgent surgical debridement

12. A 78-year-old male is in the recovery room after an open inguinal hernia repair. His blood pressure is noted to be 70/55 mmHg, and pulse is 118 bpm. He is breathing normally. The patient has a history of hypertension for which he takes a diuretic. The operation itself was uneventful except that the nurses had difficult inserting the urinary catheter. Given the hypotension, the patient is transferred to the ICU for close monitoring. The following hemodynamic parameters are obtained: cardiac output of 10.2 L/ min (normal 5 L/min) and systemic vascular resistance of 450 dynes/sec/cm5 (normal 700-1,600 dynes/sec/ cm5). What is the next step in treatment? (A) Phenylephrine (B) Dopamine (C) Blood cultures and broad-spectrum antibiotics (D) IV bolus of lactated Ringer's (E) Norepinephrine

D IV bolus

6. Which of the following is true regarding the circulation to the testes? (A) Each testis has a singular blood supply, which arises from the aorta (B) Testicular ischemia after inguinal hernia repair is most often due to accidental transection of the testicular artery (C) An ischemic testis does not lead to antisperm anti- body formation (D) During surgery for an undescended testis, the testicular artery may need to be divided (E) Lymphatic drainage from the testis is to the inguinal lymph nodes

D during surgery, testicular artery may need to be divided (for length necessary to bring it down to scrotum) three sources: testicular artery (from aorta), cremaster artery (from inferior epigastric), deferential artery (from superior vesical artery) most common cause of testicular ischemia after inguinal hernia repair it injury to the pampiniform plexus which causes severe testicular congestion and subsequent ischemia lymphatic drainage from testicle is to periaortic nodes.

9. A 40-year-old male is hospitalized after a hip replacement. On postoperative day 3, the astute medical student notices he has a new left-sided facial droop. Cardiac examination demonstrates a regular rate and rhythm without murmurs. Further exam reveals left arm and leg weakness and numbness. In addition, he has a swollen right calf that is 3 cm larger in diameter than the left calf when measured 10 cm below the tibial tubercle. Venous duplex ultrasound shows a noncompressible right femoral vein. What test is most likely to explain the etiology of the neurologic findings? (A) ECG (B) CT of the chest (C) Factor V Leiden testing (D) Echocardiogram with bubble study (E) Duplex ultrasound of the carotid artery

D echocardiogram with bubble study (paradoxical embolism from heart) Carotid artery: too young for it

12. A 61-year-old man comes to the physician with an 8-month history of increasing bilateral calf pain during his morning walks. The pain disappears with rest. He reports that the last time he saw a doctor was 20 years ago. He has a 30 pack year smoking history, but recently quit. His pulse is 68/min, respirations are 18/min, and blood pressure is 126/76 mmHg. His total cholesterol is 320 mg/dL. His doctor recommends that the patient starts a statin and aspirin to reduce the risk of adverse cardiac events. In addition, he recommends that the patient begins an exercise walking program. What would the addition of this intervention provide for the patient? (A) It will improve his overall cardiac health but will not improve his walking distance (B) It would be expected to result in an increase in his ankle-brachial index (C) It is less effective than administering a vasodilator (D) It can result in a doubling of his walking distance (E) It is contraindicated

D it can result in doubling of his walking distance

3. A 14-year-old boy is diagnosed with a varicocele by his family physician. Which of the following is true about this condition? (A) It affects either side with similar frequency (B) It causes the testicle to ride high in the scrotum (C) It transilluminates on physical exam (D) It is related to impaired venous drainage (E) It is associated with an absent cremasteric reflex

D it is related to impaired venous drainage

12. Which of the following is true of Boerhaave syndrome? (A) It most often presents with UGI bleeding (B) It is most often caused by an iatrogenic injury (C) It is a partial thickness tear of the esophagus (D) It is the most lethal GI perforation (E) Treatment consists of bowel rest and intravenous antibiotics

D it is the most lethal GI perforation

29. A 30-year-old male presents to the ED with a GSW to the right chest, just above his right nipple. In the ED, he complains of shortness of breath and severe right chest pain. His blood pressure is 110/70 mmHg, heart rate is 100/min, and respiratory rate is 20/min. On physical examination, his breath sounds are slightly diminished on the right. The trachea is midline. Neck veins are flat. The abdomen is non-tender. Upper extremity pulses are equal. A chest x-ray demonstrates a moderate right hemo- and pneumothorax. The bullet is seen in the upper chest. A chest tube is inserted into the right chest with an immediate output of 500 cc of dark blood, and after which the bleeding appears to slow down. What is the next step in the management? (A) Exploratory right thoracotomy (B) Video-assisted thoracoscopic surgery (VATS) (C) Admit to ICU for observation (D) Repeat chest x-ray (E) CT of the chest and abdomen

D repeat chest xray

1. A 25-year-old man arrives to the ED following a MVC with multiple abrasions on his abdomen. His blood pressure is 90/60 mmHg, and his pulse is 120/min. After a primary survey, a FAST exam is performed. Which of the following is a FAST exam poor at detecting? (A) Pericardial effusion (B) Single pneumothorax (C) Free peritoneal fluid in the hepatorenal space (Morrison's pouch) (D) Retroperitoneal fluid (E) Free peritoneal fluid in the perisplenic space

D retroperitoneal fluid

3. What is the most common ECG finding in a patient with PE? (A) Right ventricular strain (B) Right-axis deviation (C) S1Q3T3 (D) Sinus tachycardia (E) Right bundle branch block

D sinus tach

15. A 50-year-old man arrives to the ED with severe acute colicky flank pain and hematuria. The patient has a long-standing history of gout. Which of the following is true regarding the type of kidney stone the patient likely has? (A) Most are radiopaque (B) They are often seen in patients with hyperparathyroidism (C) Shock wave lithotripsy is not helpful (D) Sodium bicarbonate administration is beneficial (E) Suppressive antibiotics are helpful in prevention

D sodium bicarbo administration beneficial (increase urinary pH) gout stones are radiolucent

Differential for lower GI bleeding

Diverticulosis Neoplastic (rectal adenocarcinoma) Iatrogenic (colonoscopy) Inflammatory (colitis- ischemic, infectious, Anorectal (fissure, abscess (hx of Crohn's), hemorrhoids, ulcers, rectal varices) Angiodysplasia

9. A 55-year-old male comes to the physician complaining of bilateral leg pain with walking that is relieved by rest. He has a history of hypertension, hyperlipidemia, chronic obstructive pulmonary disease, and a 30-pack- year smoking history. On physical exam, his blood pressure is 139/79 mmHg, temperature is 99.3 °F, pulse is 89/min, and respirations are 16/min. Present behind both knees are small pulsatile masses. What is most strongly associated with this finding? (A) Aortic dissection (B) Hypercoagulable state (C) Degenerative joint disease (DJD) (D) Abdominal aortic aneurysm (AAA) (E) Thoracic aortic aneurysm

Dx: Popliteal artery aneurysm D abdominal aortic aneurysm

14. A male infant presents with a mass in his scrotum that has been present since birth. His mother states that the mass changes in size when the infant cries and does not seem to be painful. On exam, the mass appears to be adjacent to his testicle which is palpated posteriorly. The mass is soft and non-tender and transilluminates. There is no bulge or mass at the internal ring. What is the most appropriate next step in the management? (A) Surgical management (B) Medical management (C) Ultrasound (D) Needle aspiration (E) Reassurance

Dx: hydrocele E Reassurance Surgical repair after 1 year of life

4. A 32-year-old newlywed man presents to the ED with intense pain in his penis. He reports having an accident falling in the bathroom. His temperature is 98.8 °F, blood pressure is 126/80 mmHg, and pulse is 110/min. His physical examination is significant for a blue discoloration at the base of a swollen and deviated penis. He has no ulcers or discharge. Retrograde urethrogram did not identify any urethral injuries. What is the best next step for this patient? (A) Abdominal and pelvic CT scan (B) Cystoscopy (C) Analgesics and bed rest (D) Surgical repair (E) Temporary suprapubic catheter

Dx: penile fracture D surgical repair prior to surgery, urethral injury should be ruled out with a retrograde urethrogram.

25. A 68-year-old male comes to the physician's office for a nonhealing ulcer in his medial malleolus that has been present for several months. He has a past medical history significant for diabetes and hyperlipidemia. On physical exam, his temperature is 98.9 °F, pulse is 70/min, blood pressure is 133/79 mmHg, and respirations are 16/min. Physical exam is significant for pitting right leg edema up to the knee. The leg is warm, and the skin is shiny and has a reddish-brown appearance with several enlarged surface veins. The ulcer has granulation tissue without purulence. Dorsalis pedis pulses are 2+ bilaterally. Which of the following is most likely to assist in healing the ulcer? (A) A compressive dressing impregnated with zinc oxide (B) Diuretics (C) A custom-fitted diabetic shoe (D) Heparin (E) Oral antibiotics

Dx: venous stasis ulcer A compressive dressing impregnated with zinc oxide (cornerstone of venous stasis ulcer treatment is compression therapy!) pooling of blood leads to capillary damage and activation of inflammatory process. ulceration likely a combination of leukocyte activation, endothelial damage, and intracellular edema.

32. Which of the following clinical scenarios is the best indication for four-compartment lower extremity fasciotomies? (A) After a crush injury with an open tibia and fibula fracture (B) After successful revascularization of a leg that was ischemic for 6 h (C) After repair of a femoral artery injury due to a GSW to the leg (D) After repair of a combined popliteal artery and vein injury due to a gunshot injury (E) After an electrical burn, with a tensely swollen and tender leg and numbness in the first web space

E after electri burn

9. A 45-year-old male with a history of alcohol abuse presents to the emergency department with an UGI bleed after a night of binge drinking. The patient reports that he repeatedly dry heaved, after which he began to note bright red blood in the vomitus. He is afebrile with normal vital signs. Upper endoscopy reveals a partial tear in the mucosa and submucosa of the stomach near the gastroesophageal junction. Which of the following is true about this condition? (A) It is usually related to portal hypertension (B) It is often associated with a left pleural effusion (C) It is associated with H. pylori infection (D) The bleeding is likely arising from the gastroduodenal artery (E) The bleeding most often stops spontaneously

E bleeding often stops spontaneously

5. A 61-year-old obese female with a past medical history of diabetes undergoes laparoscopic cholecystectomy for acute cholecystitis. The operation is technically difficult and is converted to an open cholecystectomy. Twelve hours later, the patient complains of severe pain in the wound. She has a temperature of 102.8 °F, heart rate of 120/min, and a blood pressure of 110/70 mmHg. She appears ill. There is grayish foul-smelling drainage coming from the wound, which appears erythematous, swollen, indurated, and tender to touch. What is the next step in the management? (A) Reassure patient that wound infections do not occur so soon after surgery (B) Open a few of the wound staples to allow drainage (C) Broad-spectrum antibiotics (D) Return to OR for reclosure of the fascia (E) Broad-spectrum antibiotics and return to OR for aggressive wound debridement

E broad spectrum antibiotics and return to OR for aggressive wound debridement

28. A 58-year-old intoxicated homeless man arrives to the ED after getting struck by an auto. His blood pressure on arrival was 98/55 mmHg with a pulse of 120/min. Following fluid resuscitation, his blood pressure increases to 120/70 mmHg, and pulse decreases to 80/min. His abdomen is distended and mildly tender, and he has no obvious source of blood loss. A CT scan of the abdomen and pelvis shows no intraperitoneal fluid, but demonstrates bilateral pelvic fractures and a large pelvic fluid collection adjacent to the fracture with a contrast blush within it. What is the best next step in management? (A) Military anti-shock trousers (MAST) (B) External pelvic fixation (C) Open reduction, internal fixation of pelvic fracture (D) Exploratory laparotomy (E) Emergency angiography with embolization

E emergency angiography w/ embolization Open reduction, internal fi xation (C) is the defi nitive treatment for a pelvic fracture. But given the technical diffi culty and long length of such an operation, it is not recommended acutely, and especially not in someone who is actively bleeding. Pelvic packing is emerging as an alternative to angiography for pelvic bleeding.

8. A 30-year-old male arrived via paramedics after getting struck in the abdomen by a golf cart while vacationing with his family. He had no head trauma and only complained of mild abdominal pain. His vitals were normal and stable. A CT scan revealed no abnormal findings, and he was discharged on the same day. Three days later, he comes back to the ED complaining of fevers, nausea, poor appetite, and abdominal pain. A repeat CT scan shows a laceration at the neck of the pancreas with disruption of the pancreatic duct. What is the best next step in management? (A) Order serum amylase (B) Endoscopic retrograde cholangiography (ERCP) (C) CT-guided drainage (D) Magnetic resonance cholangiopancreatography (MRCP) (E) Exploratory laparotomy

E exploratory laparatomy

19. A 60-year-old man is recovering in the ICU after being rescued from a fire within a restaurant kitchen. He was trapped for a prolonged period of time. He received deep partial and full-thickness burns in over 30 % of his body. He has a past medical history of psoriasis controlled with topical steroids. On the seventh postoperative day, he becomes confused. His temperature is 96.1 °F, blood pressure is 98/72 mmHg, and pulse is 122/min. His burn wounds have focal areas with a brown color. His laboratory examination demonstrates a white blood count of 14.7 × 103/μL (normal 4.1-10.9 × 103/μL) and a serum glucose of 250 mg/dL. What is the most likely etiology for this patient's acute condition? (A) Intercompartmental fluid shift (B) Adrenal insufficiency (C) Alcohol withdrawal (D) Carboxyhemoglobinemia (E) Inflammation

E inflammation SEPSIS as defined by SIRS <98.6, >100.4 Leukocytosis >12, <4 tachycardia 90 RR >20

6. A 29-year-old female who is 8months pregnant presents with symptoms and signs of acute cholecystitis and undergoes laparoscopic cholecystectomy uneventfully. However, she returns to the ED on postoperative day 4 with acute onset dyspnea and pleuritic chest pain. A pulmonary embolism is subsequently diagnosed. What is the most likely source of the thromboembolism? (A) The right common iliac vein (B) The left common iliac vein (C) The right popliteal vein (D) The right axillary vein (E) The left femoral vein

E left femoral vein

17. A 51-year-old male is brought to the ED by paramedics following a high-speed MVC. In the ED, his blood pressure is 120/70 mmHg, and heart rate is 100/min. Hemoglobin/hematocrit is 12 g/dL (13.2-16.2 g/dL) and 36 % (40-52 %), respectively. A CT scan shows a ruptured spleen, and he is taken urgently to the operating room. During surgery, the patient is hemodynamically stable and undergoes a splenectomy. No other injuries are found, and he does not require blood transfusion. Four hours postoperatively, the patient's blood pressure drops to 80/50 mmHg, heart rate is 120/min, and urine output, which was 50 cc/h for the first 2 h after surgery, is 10 cc/h for the past 2 h. The patient is awake and only complains of thirst. He appears pale. Breath sounds are clear. Despite two liters of IV fluids, the blood pressure remains 80/50 mmHg. Repeat hemoglobin/hematocrit is 10 g/dL and 30 %. What is the next step in the management? (A) CT scan (B) Diagnostic peritoneal lavage ( C) 12-lead ECG (D) Chest x-ray (E) Return to operating room

E return to OR

11. At a follow-up appointment 2 weeks after undergoing left carotid endarterectomy (CEA), it is observed that the patient's tongue deviates to the left when he is asked to stick his tongue out. Which of the following is the most likely explanation? (A) Perioperative stroke of the medulla (B) Hematoma compressing the musculature of the oropharynx (C) Injury to cranial nerve VII (D) Injury of cranial nerve X (E) Injury of cranial nerve XII

E. Injury of cranial nerve XII

25. A 65-year-old former firefighter arrives for follow-up for chronic wound in his right leg from a burn he suffered 25 years earlier. The wound has failed to heal despite repeat skin grafting. Recently, the wound has become more painful and larger, measuring 2×2 cm, and continuously drains. Multiple biopsies of the wound are taken. Which of the following is the most important contributing factor to this patient's presenting condition? (A) Radiation exposure (B) Excess UV light exposure (C) Work-related exposure to heavy metals (D) Genetic predisposition (E) Chronic inflammation

E: chronic inflammation (Marjorlin's ulcer) squamous cell carcinoma from chronic non-healing wound

Mackler's Triad Boeerhave Syndrome

Emphysema around sternal Thoracic pain Vomiting Aggravated by swallowing

1. A 67-year-old male smoker with a known abdominal aortic aneurysm (AAA) comes to the ED with the acute onset of flank pain. On physical exam, his blood pressure is 71/49 mmHg, pulse is 121/min, temperature is 98.9 °F, and respirations are 22/min. His abdominal exam reveals distention, a pulsatile abdominal mass, and diffuse tenderness without rebound. What is the most appropriate next step in management? (A) CT scan of the abdomen (B) Exploratory laparotomy (C) Transfusion of packed RBCs (D) Abdominal ultrasound (E) Diagnostic peritoneal lavage

Ex lap (B) urgent ruptured AAA and unstable!

Supracondylar fracture

Fracture of the distal humerus

24. A 7-year-old boy presents to his pediatrician with a tense, painful, weak, and shortened forearm with a claw- like deformity of the hand. The mother states that 1 year earlier, the child fell backwards on his outstretched hand and suffered a supracondylar fracture that was treated with closed reduction and casting. The most likely explanation for the current physical exam findings is: (A) Nerve entrapment (B) Suppurative tenosynovitis (C) Ischemia/necrosis of forearm muscles (D) Complex regional pain syndrome (E) Improperly reduced fracture

Ischemia/necrosis of forearm muscles (C) Volkmann's contractures

13. A 7-year-old boy presents to the doctor with a non-tender scrotal mass. His vitals are normal and stable and he has no other complaints. His mother reports that he has been behaving more aggressively at home and has gotten into two fights at school over the past month. Physical examination reveals a firm 2.5-cm mass within the right testicle. There are no epididymal masses bilaterally, although both testicles appear to be enlarged. He also has axillary hair. Laboratory analysis reveals a normal urinalysis. What is the most likely etiology of the testicular mass? (A) Leydig cell tumor (B) Sertoli cell tumor (C) Seminoma (D) Teratoma (E) Juvenile granulosa cell tumor

Leydig cell tumor (A) -- high androgen production (Sertoli- clinically silent)

31. Which of the following parameters would be most con- sistent with acute carbon monoxide poisoning? (A) Hemoglobin decreases (B) PaO2 decreases (C) Oxygen content of blood decreases (D) Oxidized hemoglobin increases (E) Increased alveolar ventilation

O2 content of blood decreases (C) Acute carbon monoxide (CO) poisoning affects the organs with the highest oxygen demand fi rst. Patients will present in the early stages with neurologic complaints (e.g., headaches, dizziness, confusion) and cardiac symptoms (e.g., chest pain, arrhythmias). All these patients should be started on 100 % oxygen via nonrebreather facemask. CO has nearly 250× more affi nity for hemoglobin than oxygen. Thus the hemoglobin- oxygen dissociation curve shifts to the left, and more hemoglobin is bound by CO than it is by oxygen. This decreases both the hemoglobin saturation (of oxygen) and the oxygen content in the blood.

12. Burn patients are at risk for multiple infections. What is the most common organism to cause infection in burn patients? (A) Staphylococcus aureus (B) Streptococcus pyogenes (C) Streptococcus agalactiae (D) Pseudomonas aeruginosa (E) Candida albicans

Pseudomonas aeruginosa

May Thurner Syndrome

Right common iliac artery compresses left common iliac vein Therefore, DVTs occur more commonly in left leg

23. A 65-year-old male carpenter states that his left arm gets tired when he uses it at work, forcing him to stop and rest. In addition, he notes that when using his left arm, he experiences dizziness and vertigo. He has a long-standing history of smoking. Physical examination reveals normal brachial and radial pulses on the right but markedly decreased brachial and radial pulses on the left. In addition, there is an audible bruit just above his left clavicle. Which of the following is true about this condition? (A) The dizziness and vertigo are due to blood being diverted from the anterior brain circulation (B) Stenting of the left subclavian artery may be helpful (C) Systolic blood pressure measurements in right and left arms are likely to be the same (D) It is most likely due to an inflammatory arteritis (E) It more commonly affects the right arm

Stenting of the left subclavian artery may be helpful (B) steals blood from the posterior circulation, as blood travels in reverse fashion down vertebral artery

23. A construction worker is digging a trench when he cuts his arm on a rusty nail in the soil. He is 45 years old and has not been to the doctor since he was a teenager, but he is confident he received all of his vaccinations up to age 18. What is the next step in treatment? (A) Tetanus immunoglobulin only (B) Tetanus vaccination only (C) Tetanus immunoglobulin and vaccination (D) Primary wound closure (E) Clindamycin for 3 weeks

Tetanus vaccination only Clean wounds <3 doses: give tetanus vaccine >3 doses: administer the vaccine only if they haven't received in the past 10 years Dirty wounds: <3 doses: give tetanus vaccine and Ig >3 doses: give vaccine if last dose was greater than 5 years ago

26. A 18-year-old man arrives to the ED combative and with severe shortness of breath, after suffering a stab wound to the chest. His blood pressure is 94/76 mmHg with a pulse of 120/min and respiratory rate of 28/min. Physical exam reveals a 2 cm stab wound on the left chest. Lung fields on the left have decreased breath sounds and are hyperresonant to percussion. His neck veins are dis- tended. A needle is placed in the left second intercostal midclavicular line and aspirated until a gush of air is heard escaping the chest wall. A liter of normal saline is given, and blood pressure improves to 120/70 mmHg, and pulse decreases to 100/min. What is the best next step in management? (A) Transfuse O-negative blood (B) Tube thoracostomy (C) Chest x-ray (D) CT scan of the chest (E) Transport to the operating room for a thoracotomy

Tube thoracostomy B

10. A 62-year-old man presents to the physician with swelling in both legs that has been going on for several months. He has a past medical history significant for hypertension and hyperlipidemia. He states that the swelling becomes progressively worse throughout the day. He denies any pain associated with the swelling. Physical exam reveals a temperature of 99.1 °F, blood pressure of 125/72 mmHg, pulse of 77/min, and respiratory rate of 16/min. Heart is in regular rate and rhythm with no murmurs. JVP is 7 cm. The lungs are clear to auscultation. On extremity exam, there are strong posterior tibialis and dorsalis pedis pulses bilaterally. The legs are edematous up to the knee with a small patch of eczematous, erythematous-brown scaling skin rash located on the medial malleolus of the right ankle. Which of the following is the most likely cause of his edema? (A) Peripheral artery disease (B) Deep venous thrombosis (C) Renal protein loss (D) Reduced cardiac output (E) Valve incompetence

Valve incompetence (E) hemosiderin deposition!!

6. A 52-year-old male is seen at the ED for the acute onset of swelling in his right leg that began 6 h ago. He has a past medical history significant for hypertension and colon cancer being treated with chemotherapy in the outpatient setting. Physical exam reveals a temperature of 99.1 °F, blood pressure of 125/72 mmHg, pulse of 77/ min, and respiratory rate of 16/min. On extremity exam, there are 2+ posterior tibialis and dorsalis pedis pulses bilaterally. The right leg is edematous, warm, and markedly more swollen than the left leg. What is the best initial study in the work-up? (A) Ankle-brachial index (ABI) (B) Venous duplex scan (C) CT angiogram of lower extremities (D) CT of the head (E) Measurement of compartment pressures

Venous duplex scan (B) DVT

4. What is the most common initial acid/base abnormality seen in patient with a PE? (A) Respiratory alkalosis (B) Respiratory acidosis (C) Metabolic alkalosis (D) Metabolic acidosis (E) Combined respiratory alkalosis and metabolic acidosis

a respiratory alkalosis (hypoxia -> hyperventilation -> hypocapnia -> alkalosis) INITIALLY later compensated with metabolic acidosis

16. A 45-year-old patient undergoes a hernia repair. He is a smoker. On postoperative day 2, his wound appears to be healing well, and he is discharged home. What type of operative wound is this considered to be? (A) Clean (B) Clean contaminated (C) Contaminated (D) Dirty infected (E) Elective

a. clean

17. A 69-year-old immigrant from Mexico comes into the hospital with a 2-day history of severe pain in his left calf. He took Vicodin that a neighbor gave him with only minimal relief. Over the last day, he has lost sensation and movement in his left foot, which prompted him to finally come to the hospital. He has had no medical care and has no past medical history. Physical exam reveals a temperature of 98.5 °F, blood pressure of 147/79 mmHg, heart rate of 85/min, and respiratory rate of 16/min. On extremity exam, there are absent popliteal, posterior tibialis, and dorsalis pedis pulses on the left. Sensation to pinprick is absent in the left, and he cannot plantar- or dorsiflex his foot. The skin of the foot appears to be mottled. Doppler of the foot vessels reveals no audible signals. What is the definitive treatment? (A) Catheter-assisted thrombolysis (B) Open surgical intervention (C) Amputation (D) Aspirin (E) Intravenous heparin

acute left calf no sensation/movement -> either IIb or III why III? mottled foot, 48 hrs duration of ischemia amputation (C)

Rinner test: - sensorineural hearing loss:

air conduction is heard longer than bone conduction (like normal, but just not as twice as long)

Horner's syndrome

anhidrosis, ptosis, meiosis may suggest injury to common carotid and internal carotid arteries because the sympathetic fibers that innervate the pupil, eyelid, and skin run along those.

shoulder girdle dislocation- artery involved and features?

axillary posterior dislocation (seizures- axillary nerve inuries) anterior dislocation (more common- axillary artery injuries)

14. A 60-year-old female returns to the ED for right leg swelling 1 week after undergoing a right hemicolectomy for cecal adenocarcinoma. Duplex scan confirms a DVT and the patient is started on IV heparin. The patient's hospital course is further complicated by UTI and pneumonia. On hospital day 7, the morning CBC shows a platelet count of 55,000, down from a baseline of 140,000. What is the next immediate step? (A) Stop heparin (B) Stop heparin and start a direct thrombin inhibitor (C) Stop heparin and switch to low molecular weight heparin (D) Transfuse 2 units of platelets (E) Start corticosteroid

b. stop heparin and start a direct thrombin inhibitor (argatroban) not LMWH because it can still have HIT effects

Rinner test: - conductive hearing loss:

bone conduction is heard longer than air conduction (bone conduction: mastoid process tuning fork) (air conduction: tuning fork next to ear)

supracondylar- artery involved and features?

brachial children, Volkmann's contracture

pelvic- artery involved and features?

branches of internal iliac (superior gluteal and internal pudendal) posterior fracture- superior gluteal anterior fracture- internal pudendal

hip dislocation- artery involved and features?

femoral artery posterior dislocation: internal rotated and adducted leg anterior dislocation: externally rotated abducted leg

radial nerve entrapment

more common with middle and distal third humerus fractures triceps function remains intact distal loss of function: wristdrop, loss of metacarpophalangeal joint extension, sensory loss over dorsum of hand

odynophagia

pain with swallowing suggestive of injury to oropharynx or esophagus

combined stroke + DVT

paradoxical embolism

tibial plateau fracture

popliteal artery medial injuries more common to cause arterial injuries than lateral

knee dislocation

popliteal artery posterior dislocation injuries more common to cause arterial injuries than anterior

stridor

sign of upper airway obstruction caused by compression of trachea from a large hematoma/soft tissue swellig/direct laryngeal injury/bilateral recurrent laryngeal nerve injry ET TUBE!!!!!!! EMERGENT!!!

clavicle- artery involved and features?

subclavian associated pneumothorax/hemothorax


Set pelajaran terkait

CompTIA Security+ SYO 601 Chapter 6 Secure Coding

View Set

California Life Practice Exams A&B (old version)

View Set

Chapter 5: Fraud, Internal Control, and Cash

View Set

Intro to Law midterm study guide

View Set

Nursing Management: Patients With Immunodeficiency, HIV Infection, and AIDS

View Set